At birth the blood volume is approximately
A. 65 mL/kg body weight
B. 85 mL/kg body weight
C. 110 mL/kg body weight
D. 125 mL/kg body weight
E. 150 mL/kg body weight
At birth the blood volume is approximately
A. 65 mL/kg body weight
B. 85 mL/kg body weight
C. 110 mL/kg body weight
D. 125 mL/kg body weight
E. 150 mL/kg body weight
(B) at birth, the blood volume is about 85 mL/kg body weight, falling to about 75 mL/kg after the first month.
A previously healthy full-term infant has several episodes of duskiness and feeding difficulties during the second day of life. She is noted to have increasing jaundice. Which of the following tests will be the least helpful in making a diagnosis?
A. Cerebrospinal fluid
B. Urine culture
C. Total bilirubin
D. Endotracheal aspirate cultures
E. Venous blood
(D) for neonates isolation of normally sterile fluid from cerebrospinal fluid, urine, and venous blood will help in identifying the bacterial agent responsible for infection. In children less than 2 months with temperatures of 38oC a complete evaluation includes a lumbar puncture, chest X-ray, urinalysis, and blood culture when the child has no focal signs. An elevation of total bilirubin causing jaundice is associated with sepsis or urinary tract infection.
The Apt test is used for what purpose?
A. Crude test for carbon monoxide poisoning
B. Semiquantitative test for lead poisoning
C. Qualitative test for fetal hemoglobin
D. Screening test for S hemoglobin
E. Test for blood viscosity
(C) The Apt test is useful in distinguishing maternal blood from newborn blood.
Polycythemia in the newborn is characterized by all of the following except:
A. increased incidence in infants of diabetic mothers
B. placental transfusion decreased by late clamping of the umbilical
cord
C. clinical presentation that includes jitteriness and poor feeding
D. complications including heart failure and necrotizing enterocolitis
E. elevated viscosity of whole blood
(B) Polycythemia is seen in infants of diabetic mothers and for whom there was late clamping of the umbilical cord. Neurologic symptoms include jitteriness and poor feeding and appear with plethora and acrocyanosis. Consequences include heart failure and necrotizing enterocolitis. Elevated viscosity occurs at hematocrits above 65%. It is accentuated by the decreased deformability and filterability of neonatal red blood cells predisposing to stasis in small capillaries.
For a newborn infant, the least important factor to consider in assessing the risk for kernicterus is:
A. breast-feeding
B. acidosis
C. sepsis
D. albumin levels
E. cephalosporin antibiotics
(A) While breast-feeding can cause jaundice, no cases of kernicterus have occurred in healthy breast-feeding infants. Acidosis and sepsis along with asphyxia, hypoglycemia, respiratory distress syndrome, intraventricular hemorrhage, and meningitis predispose an infant to kernicterus. Since bilirubin is bound to albumin, decreased albumin levels or loss of binding sites through displacement by drugs increases the amount of bilirubin that may enter the brain
Fetoprotein levels from maternal serum may be helpful in diagnosing
A. congenital cardiac malformations
B. prune-belly syndrome
C. cleft lip and palate
D. Down syndrome
E. fetal alcohol syndrome
(D) Detection of low levels obtained from maternal reported a higher incidence of trisomy 21. Low levels also accompany increased incidence of fetal growth disturbance, fetal hypoxia, and other poor pregnancy outcomes. Elevated levels obtained at 16 to 18 weeks gestation identify 80 to 90% of fetuses at risk for anencephaly and meningomyelocele. When the level is greater than 2.0 times the median value, an ultrasound and amniocentesis are used to examine the fetus.
Mongolian spots are characterized by all the following except
A. they are permanent
B. they are usually of a slate blue pigmentation
C. they are generally observed over the buttocks
D. the area of pigmentation is well demarcated
E. they are not associated with trisomy syndromes
(A) Mongolian spots usually disappear within the first year of life. There is no increased incidence of these lesions with the various trisomy syndromes.
All the following physical signs may be useful in estimating gestational age at birth except
A. there are only one or two transverse skin creases on the sole of
the foot until 36 weeks of gestation
B. the breast nodule is usually not palpable at 33 or 34 weeks
C. the breast nodule is usually 4 to 10 mm in term infants
D. the testes are descending and rugae cover the entire scrotal
surface by 34 weeks
E. the texture of scalp hair
(D) The tests are usually not completely descended until after 36 weeks and scrotal rugae are few and limited to the anterior and inferior aspects of a relatively small scrotum.
Caput succedaneum is characterized by all the following except
A. a diffuse, edematous swelling of the soft tissues of the scalp,
involving the portion presenting during vertex delivery
B. it may extend across the midline
C. it may extend across suture lines
D. edema usually disappears within 2 to 3 months
E. the scalp overlying the area may show mild bruising
(D) The edema of caput succedaneum usually disappears within the first few days of life and requires no specific therapy.
A depressed infant is transferred to the nursery intubated and hand ventilated. Blood gas shows Po2: 225 mmHg, Pco2: 38 mmHg, pH: 7.25. The infant is pink and active. Suddenly she turns blue, drops her heart rate, and has decreased breath sounds on the left. You should:
A. needle and aspirate the left chest
B. insert an umbilical venous catheter and administer a pressor agent
C. slowly pull back on the endotracheal tube and auscultate both
sides of the chest
D. begin an infusion of 2 mEq of NaHCO3/kg
E. begin external cardiac massage
(C) The first step is to consider that the tube has slipped down the right mainstem bronchus. This is easy to remedy. Pneumothorax is certainly a consideration but should not be the first one should consider in this situation.
In a newborn with oral moniliasis, the most common primary source of infection is
A. maternal source (vaginal)
B. contaminated fomites
C. following use of AgNO3
D. contact by hospital carriers
E. systemic antibiotic therapy
(A) There is a positive correlation between maternal vaginal and infantile oral moniliasis. This is the primary means of infection in a newborn.
Meconium impaction is associated with
A. cretinism
B. cystic fibrosis
C. thrush
D. hyaline membrane disease
E. trisomy 21 syndrome
(B) Meconium ileus is associated with cystic fibrosis. Deficiency of pancreatic enzyme limits normal digestive activities in the intestine. As a result, meconium is left in a viscid mucilaginous state, and it clings to the walls of the intestine. Movement is difficult or impossible.
Persistent jaundice during the first month of life may be associated with all the following except
A. cytomegalic inclusion disease
B. congenital atresia of the bile ducts
C. galactosemia
D. Rh incompatibility
E. penicillin treatment
(E) All but penicillin therapy has been associated with persistent jaundice during the first month of life. Cytomegalic inclusion disease causes a hepatitis, hepatosplenomegaly, and a jaundice from conjugated hyperbilirubinemia. Congenital atresia results in accumulation of conjugated bilirubin associated with the obstruction of or reduced numbers of bile ducts. Galactosemia resulting from the absence of galactose-1-phosphate uridyl transferase causes injury to the liver from the accumulation of galactose. It persists until the initiation of a glactose free diet. Finally, a sequela of hemolytic disease is the significant elevation of direct and indirect bilirubin causing a disorder called inspissated bile syndrome. The jaundice clears within a few weeks or a month.
All the following are characteristic of jaundice associated with breast-feeding except
A. significant elevations of unconjugated bilirubin
B. a rapid fall in serum bilirubin after discontinuation of nursing
C. nursing can be resumed after several days without return of
hyperbilirubinemia
D. significant elevations of conjugated bilirubin
E. kernicterus has never been reported to occur as a result of breast
milk jaundice alone
(D) Direct bilirubin is not elevated. Unconfirmed evidence suggests that some mothers have substances in their milk that competitively inhibit glucoronyl transferase conjugation. Other sources identify the relatively low fluid and calorie intake in the first week as the etiology. The management should emphasize frequent feedings 2 to 3 hours apart day and night to increase the amount of fluid and to stimulate stooling with elimination of bilirubin from the intestines. The rise in bilirubin by the third day of life is closely associated with breast-feeding. Kernicterus has not been seen in healthy, jaundice breast-fed infants.
All the following are usually associated with cretinism except
A. constipation
B. prolonged jaundice
C. lethargy
D. tetany
E. hypotonia
(D) Neonatal tetany characterized by irritability, poor feeding, vomiting, and occasionally seizures is usually associated with transient hypoparathyroidism and not thyroid deficiency.
Hypoglycemia has been observed in newborns
A. with low birth weight and respiratory distress syndrome
B. with anoxic injury
C. with hypothermia
D. who are small for gestational age
E. with high Po2
(E) Newborns with severe illnesses may develop hypoglycemia as a result of increased metabolic needs that are out of proportion to substrate stores. Hyperoxia by itself is not a stimulus for hypoglycemia.
Directions: The set of lettered headings below is followed by a list of numbered words or phrases. For each numbered word or phrase select:
A. if the item is associated with A only
B. if the item is associated with B only
C. if the item is associated with both A and B
D. if the item is associated with neither A nor B
In the first 3 days of postnatal life, which could rise?
A. Weight
B. Bilirubin
C. Both
D. Neither
(B) Weight loss occurs during the first 3 to 4 days of life. Jaundice may be present at birth or rise at any time in the neonatal period depending on the condition producing the elevation in bilirubin level.
In the first 3 weeks in the breast-fed infant, which could rise?
(C) The course of breast-feeding jaundice can have prolonged elevation of bilirubin levels through the third week of life. Breastfed infants should regain their birth weight by 14 days of life compared to 10 in the formula fed infant.
A 2-day-old infant presents to the emergency room with signifi-cant bleeding from the rectum and nose. He was the product of a full-term pregnancy that appeared to be without complications. He was delivered at home by a lay midwife and had Apgar scores of 9 and 9 at 1 and 5 minutes, respectively. He had been doing well on breast-feedings; this is his first visit to a health care pro-fessional since birth. Which of the following vitamin deficiencies might explain his condition?
A. Vitamin A
B. Vitamin B
C. Vitamin C
D. Vitamin D
E. Vitamin K
E. new born infants have a relative deficiency of vitamin K, especially those who are breast-fed; most infants are routinely administrated vitamin K at birth to prevent bleeding complications associated with deficiency.
A term infant is delivered vaginally to a 22-year-old gravida 2, para I mother. Immediately after birth the child is noted to have a scaphoid abdomen, cyanosis, and respiratory distress. Heart sounds are heard on the right side of the chest, and the breath sounds seem to be diminished on the left side. The next step in resuscitation in this child is to:
A. Initiate bag and mask intubation.
B. Administer intravenous naloxone.
C. Administer bicarbonate intravenously.
D. Intubate with an endotracheal tube.
E. Initiate chest compressions immediately.
D. The case describes a child with diaphragmatic hernia. As a result of herniated bowel contents into the chest, children with this condition often have pulmonary hypoplasia. Bag and mask ventilation will cause accumulation of gas into the bowel (which is located in the chest) and further respiratory compromise. Therefore, endotracheal intubation is the best course of action
A 37-week gestation infant is born to a 33-year-old gravida 3 mother. The pregnancy was uncomplicated, but the infant was noted at birth to be lethargic and with a slow heart rate. Oxygen was administered via bag and mask and the child was intubated, but the infant’s heart rate remained at 40 bpm. The most appropriate next step is to:
A. Begin chest compressions.
B. Administer intravenous bicarbonate.
C. Administer intravenous atropine.
D. Administer intravenous epinephrine.
E. Administer intravenous calcium chloride.
A. If the heart rate is still below 60 bpm despite positive pres¬sure ventilation with 100% oxygen, then chest compressions are used for 30 seconds. If the heart rate is still below 60 bpm, then drug therapy (usually epinephrine) is indicated.
A term infant is born by vaginal delivery after an uncomplicated pregnancy. The child appears to be normal but develops respiratory distress each time he stops crying. When crying, he is pink. When he is not crying, he makes vigorous respiratory efforts but becomes dusky. The likely explanation for this child’s symptoms is:
A. Choanal atresia
B. Diaphragmatic hernia
C. Neonatal narcosis
D. Meconium aspiration
E. Pneumothorax
A. Infants are obligate nose breathers. When crying they can breathe through their mouth, but when quiet they must have a patent nose. Choanal atresia is demonstrated by attempting to pass a feeding catheter through each nostril. When choanal atresia is diagnosed by failure to pass the feeding tube, endo¬tracheal intubation to bypass the airway obstruction is done until surgi¬cal repair can be completed.
An infant girl is born via spontaneous vaginal delivery at 28 weeks’ gestation because of an incompetent cervix. Which of the following features of her clinical course in the neonatal in¬tensive care unit is most likely to correlate with her clinical out¬come 5 years from now?
A. Grade IV intraventricular hemorrhage
B. Administration of surfactant
C. Retinopathy of prematurity stage 1 on initial ophthalmologic examination
D. Apnea of prematurity
E. Umbilical artery catheterization
A. Intraventricular hemorrhage is a frequent complication in preterm infants, and is associated with seizures, hydrocephalus and periventricular leukomalacia (PVL). A grade IV bleed in¬volves the brain parenchyma and puts this child at higher risk for neurodevelopmental handicap.
An infant is delivered via cesarean section at 35 weeks gestation because of macrosomia and fetal distress. The mother has class D gestational diabetes. Despite regular visits at a high-risk ob-stetrics clinic, her hemoglobin A1c level is 16. This infant is particularly at risk for birth asphyxia, cardiac sep¬tal hypertrophy, polycythemia, and:
A. Hyperglycemia
B. Pneumothorax
C. Hyaline membrane disease
D. Congenital dislocated hip
C. Infants of diabetic mothers, especially when poorly con¬trolled during pregnancy, are at risk for respiratory distress syndrome.
A premature infant of a Class B diabetic mother who received late prenatal care is delivered via cesarean section due to fetal distress. The mother’s axillary temperature just prior to delivery is 98.6°F (37°C). The child has poor color and tone at birth, and no spontaneous cry. Respiratory effort is minimal, and the pulse is weak and slow at 80 beats per minute. After endotracheal in-tubation, the color and tone improve somewhat, but the baby still has perioral cyanosis and the heart rate is 90 beats per minute. What is the most likely cause of this infant’s persistent distress?
A. Sepsis
B. Impaired cardiac function
C. Renal failure
D. Hypoglycemia
E. Hypocalcemia
B. Infants born to mothers with poorly controlled gestational di¬abetes are at risk for congenital heart anomalies, cardiomyopa¬thy, septal hypertrophy, and subaortic stenosis. This child’s symptoms of failure of heart rate and color to fully respond to re¬suscitation efforts and the maternal history of diabetes are clues that suggest the infant is at higher risk of cardiac problems. Sep¬sis can present in a similar fashion and certainly should remain in the differential diagnosis until further evaluation is complete, but no particular risk factors for infectious disease are known in this case. This child is also at high risk for hypoglycemia in the first hours of life, but hypoglycemia alone would be a less likely explanation for his constellation of symptoms
Which of the following factors decreases the risk of neurologic damage in a jaundiced newborn?
A. Hypoalbuminemia
B. Displacement of bilirubin from binding sites by such drugs as sulfasoxazole
C. Acidosis
D. Sepsis
E. Maternal ingestion of phenobarbital during pregnancy
E. Administration of phenobarbitol induces glucuronyl transferase in newborn infants, thus reducing rather than exacerbating neonatal jaundice
Gilbert syndrome is caused by:
A. Increased production of bilirubin
B. Impaired conjugation of bilirubin
C. Deficient hepatic uptake of bilirubin
D. Severe deficiency of uridine diphosphate glucuronosyl-transferase
E. Glucose-6-phosphate dehydrogenase (G6PD) deficiency
C. Gilbert disease is associated with abnormality of bilirubin transport in hepatocye.
A 30-hour-old full-term infant has jaundice of the face and chest. He is breast-feeding well and has a normal physical ex-amination except for the jaundice. His serum bilirubin level is 16.5 mg/dL. The best course of action is to:
A. Recommend discontinuation of breastfeeding for 48 hours and supplement with formula
B. Start phototherapy
C. Wait 6 hours and retest the serum bilirubin level
D. Start an exchange transfusion
E. No action is needed
B. Although the etiology of the hyperbilirubinemia must be investigated, phototherapy should be started.
A newborn infant was born at home. At 2 days of life he is noted to have puffy, tense eyelids, red conjunctivae, a copious amount of purulent ocular discharge and chemosis. The most likely diagnosis is:
A. Dacryocystitis
B. Chemical conjunctivitis
C. Pneumococcal ophthalmia
D. Gonococcal ophthalmia
E. Chlamydial conjunctivitis
D. The time of onset of symptoms in a neonate with conjunctivitis is somewhat helpful in the diagnosis of ophthalmia neonatorum. Chemical conjunctivitis is a self-limited condition that presents within 6 to 12 hours of birth; it is a consequence of irritation secondary to silver nitrate or erythromycin prophylaxis applied to the eyes. Gonococcal conjunctivitis usually has its onset within 2 to 5 days of birth and is the most serious of the bacterial infections; prompt and aggressive topical treatment and systemic antibiotics are indicated to prevent serious com¬plications such as corneal ulceration, perforation and resulting blindness. Parents should be treated for gonococcal disease to avoid the risk to the child of reinfection. Chlamydial conjunc¬tivitis often presents 5 to 14 days after birth and is usually treated with systemic erythromycin (in part, to reduce the risk of the in¬fant’s developing chlamydial pneumonia at 2 to 3 months of age). The benefits of using oral erythromycin in infants must be weighed with the increased risk of their developing hypertrophic pyloric stenosis, a condition found to be more common in chil¬dren having received oral erythromycin. Both parents of a child with chlamydial conjunctivitis must also be treated.
A full-term 3500-g female newborn delivered by cesarean sec-tion develops a respiratory rate of 70 breaths per minute;: and ex¬piratory grunting at 1 hour of life. She has good tone, good color, and a strong suck. The most likely diagnosis in this infant is:
A. Sepsis
B. Transient tachypnea of the newborn
C. Respiratory distress syndrome
D. Meconium aspiration syndrome
E. Tracheoesophageal fistula
B. Transient tachypnea of the newborn (TTN) is a respiratory condition that results from incomplete evacuation of fetal lung fluid in full-term infants. It is more common in cesarean deliveries. It usually disappears within 24 to 48 hours of life. Often no treatment is indicated unless the infant requires low amounts of supplemental oxygen.
An infant is born at term to a 23-year-old known HIV-positive mother. The mother has been followed closely during the preg-nancy, and she has been taking antiretroviral medications for the weeks prior to the delivery of her infant. Routine management of the healthy infant should include:
A. Admission to the neonatal intensive care unit for close car¬diovascular monitoring
B. HIV enzyme-linked immunosorbent assay (ELISA) on the infant to determine if congenital infection has occurred
C. Beginning a course of zidovudine for the infant
D. Chest radiographs to evaluate for congenital Pneumocystis carinii
E. Administration of intravenous immunoglobulin (IVIG) to the baby to decrease the risk of perinatal HIV infection
C. HIV transmission from mother to infant has decreased by more than 50% over the past 15 years, probably as a result of perinatal administration of antiretroviral medications to the mother and a course of zidovudine to the exposed infant. ELISA is an antibody test that will be positive in the infant because ma¬ternal antibodies for her HIV disease are passed through the pla¬centa; it is not a useful test in the newborn infant to determine neonatal infection. IVIG has not been shown to have a role in decreasing perinatal transmission. Healthy infants born to HIV¬ infected mothers do not need special monitoring nor do they need routine radiographs.
Which of the following infants most warrants home cardiorespiratory monitoring?
A. A healthy 3-month-old infant who was born at term and whose weight is at the 5th percentile for age.
B. A healthy infant who was born at 29 weeks gestation and whose weight is at the 50th percentile for age.
C. A 5-month-old infant with a history of recurrent bouts of wheezing.
D. A premature infant with abnormal apnea and bradycardia.
E. A healthy term infant whose older sibling died of SlDS.
D. Home cardiorespiratory monitoring has not been shown to decrease the incidence of SIDS. Therefore, the potential bene¬fits of decreased caregiver anxiety and possible avoidance of an adverse event must be considered against the inconvenience and cost involved. Symptomatic premature infants (i.e., those with apnea and bradycardia events) are among those who are consid¬ered to be at higher risk for an adverse event, and for whom monitoring is therefore recommended. Monitors are also rec¬ommended by some. but not all experts when more than one child in a family has died from SIDS. Monitoring is not recommended for the infants in choices A, B, or C.
A premature infant born at 28 weeks gestation develops necro-tizing enterocolitis and intestinal perforation requiring the re-moval of a small portion of his colon and 20 cm of small intes-tine, including the ileocecal junction. Future nutritional support will likely include:
A. Vitamin B 12 oral supplementation.
B. Long-term parenteral nutrition.
C. Vitamin BI2 injections every 1 to 3 months.
D. Liquid-only diet.
E. Supplemental oral folate.
C. The terminal ileum is the location of B12 absorption; the child in this case has had the terminal ileum resected. Oral re¬placement of vitamin BI2 would be ineffective. Folate is ab¬sorbed throughout the small intestine. The amount of resected intestine is insufficient to cause the need for long-term par¬enteral nutrition, as the infant still has an adequate amount of small intestine remaining.
A 2-week-old infant develops a temperature to 102°F (38.9°C). The infant was the product of an uncomplicated term gestation and has been at home since the second day of life. The irritable, fussy infant has a heart rate of 170 beats per minute and a res-piratory rate of 40 breaths per minute. The infant’s anterior fontanelle is full, but demonstrates no nuchal rigidity; the rest of the physical examination is unremarkable. Appropriate manage-ment of this infant is to:
A. Encourage oral fluids and office follow-up in 24 hours.
B. Prescribe oral amoxicillin and office follow-up in 1 week.
C. Perform a lumbar puncture, blood culture, urine culture, and admit to the hospital.
D. Order computerized tomography of the head followed by lumbar puncture.
E. Prescribe intramuscular ceftriaxone and office follow-up in 1 week.
C. This infant potentially has a serious bacterial infection, and a complete evaluation including a lumbar puncture is performed. Infants do not reliably demonstrate Kernig or Brudzinski signs; thus. a lack of nuchal rigidity should not preclude a lumbar puncture. Performing a computerized tomography scan before lumbar puncture in an infant with an open anterior fontanelle is rarely necessary as herniation of the brain is exceedingly rare.
You are asked to see a I-month-old infant to provide a second opinion. During a brief: self-limited and untreated diarrheal episode last week, his primary physician ordered a stool assay for Clostridium difficile toxin, which has subsequently come back positive. The infant now is completely asymptomatic, ac¬tive, smiling, and well hydrated. His physician told the mother treatment was not necessary, but the mother was concerned and is asking what you think should be done. You correctly respond:
A. Clostridium difficile commonly colonizes the intestine of infants, and treatment is not warranted in this case.
B. The infant should take a 7-day course of oral metronidazole to prevent future complications.
C. The infant should take a 10-days course of oral vancomycin to prevent future complications.
D. The infant should be admitted to the hospital for intravenous metronidazole to prevent future complications.
E. A repeat study needs to be performed to look for the Clostridium difficile organism.
A. C. difficile colonizes approximately half of normal healthy infants in the first 12 months. In this infant with no history of antibiotic treatment and no current symptoms, treatment is unnecessary. C. difficile colitis rarely occurs without a history of recent antibiotic use.
A 10-day-old infant presents to the pediatric clinic with a painful, red vesicular rash in the diaper area. He is mildly fussy but afebrile, and he has good oral intake. What is the appropri¬ate management of this infant?
A. Prescribe an antifungal cream and follow up by telephone in 24 hours.
B. Perform a Tzanck smear and send the patient home if no multinuclear giant cells or intranuclear inclusions are found.
C. Obtain specimens for HSV culture and admit to the hospital for parenteral acyclovir pending test results.
D. Schedule an appointment with a pediatric dermatologist.
E. Order a STAT EEG and brain MRI.
C. unlike older children and adults with localized cutaneous disease, infants with suspected herpes skin lesions requires par¬enteral antiviral therapy to prevent more serious sequelae.
A woman presents for her first prenatal visit at 9 weeks’ gesta-tion. She reports that she is generally healthy, except that she has an outbreak of genital herpes approximately once a year. To pre-vent transmission of the virus to her infant, her physician should:
A. Place her on oral acyclovir for the remainder of the pregnancy.
B. Perform weekly genital viral cultures starting at 36 weeks’ gestation.
C. Anticipate a cesarean section delivery.
D. Offer a cesarean delivery if herpetic lesions or prodromal symptoms are present when labor has begun.
E. No change in management is indicated, as the risk of trans¬mission to the infant is low even if she has an outbreak at the time of delivery.
D. Even though the risk of viral transmission in the setting of a recurrent outbreak of herpes is low, cesarean section is indicated if lesions are present at the time of delivery. Surveillance cul¬tures are not likely to be helpful as negative results obtained a few days prior do not preclude an outbreak at the time of deliv¬ery, and results of a more recently obtained specimen may not be available. Culture specimens may be indicated at 48 hours of life from the infant born to herpes positive mother and treat¬ment with acyclovir initiated if any culture is positive or there are signs of neonatal HSV 1
A mother brings her 2-week-old son to the clinic for a well-baby visit. Her only concern about the infant is a “rash” on his face and scalp that began approximately 1 week earlier. Examination reveals a healthy white male with normal vital signs and a nor¬mal examination except for the skin. He has yellowish, waxy¬ appearing, adherent plaques on the scalp, forehead, cheeks, and nasolabial folds. Which of the following is the most likely ex-planation for the infant’s skin findings?
A. Lichen planus
B. Tinea versicolor
C. Atopic dermatitis
D. Seborrheic dermatitis
E. Psoriasis
D. Seborrheic dermatitis presents in infancy and adolescence. The chronic symmetric eruption, characterized by overproduction of sebum, affects the scalp, forehead, retroauric¬ular region, auditory meatus, eyebrows, cheeks and nasolabial folds. The treatment includes softening the scales with mineral oil and daily shampooing with a mild shampoo; some suggest shampoos containing ketoconazole (Nizoral). Low to medium potency topical corticosteroids may be used. Avoid scrubbing the affected areas.
A 7-day-old infant is brought to clinic because he has developed “pimples” on his cheeks and forehead. The infant is breast¬feeding well and the parents have no other concerns about him. The skin around the pimples on the face has a red base yet does not appear to be secondarily infected. There are no other areas of breakouts on the body. This infant most likely has:
A. Atopic dermatitis
B. Urticaria
C. Herpes simplex
D. Neonatal acne
E. Tinea corporis
.
D. Approximately 20% of normal neonates develops at least a few comedones within the first month of life. The cause of neonatal acne is unknown but has been attributed to placental transfer of maternal androgens, hyperactive adrenal glands, and a hypersensitive neonatal end-organ response to androgenic hormones
A 2-week-old male infant presents to the clinic with his foster mother because he has bilateral purulent eye discharge. His pre-natal history is unknown. The examination demonstrates signif-icant tarsal conjunctivitis and eye discharge. Which of the fol-lowing statements is true?
A. Initial therapy includes administration of intramuscular ceftriaxone.
B. The organism most likely to be responsible for this infant’s conjunctivitis also causes pneumonia in infants aged 1 to 3 months.
C. Immediate referral to a pediatric ophthalmologist is warranted.
D. Warm compresses and gentle massage are first-line therapies.
E. Topical antimicrobial therapy is preferred.
B. A chlamydial infection is the most likely cause of conjunc¬tivitis in this 2-week-old infant. Chlamydia trachomatis causes pneumonia in infants, generally between the ages of 1 and 3 months. Infants present with cough, tachypnea, and rales and are afebrile. A complete bood count might show eosinophilia. A 2- to 3-week course of’ oral erythromycin is given.
You are called to examine a term infant who required resuscita-tion after a spontaneous vaginal delivery. The Apgar scores at 1, 5, and 10 minutes were 2, 7, and 9, respectively. The mother’s records show that she received routine prenatal care, and that her prenatal ultrasonogram, triple screen, and glucose tolerance tests were all within normal limits. The nurse tells you that the father seemed very agitated and that he mentioned possibly su¬ing the obstetrician if the baby does not “turn out normal.” Your thorough examination of the baby reveals no abnormalities. In counseling the family, which one of the following is most ap-propriate?
A. Tell them that your examination findings indicate that everything is fine.
B. Tell them that the low Apgar scores at I and 5 minutes indi¬cate that the baby suffered perinatal asphyxia.
C. Tell them that as the pregnancy was uncomplicated, and that any neurologic deficit that the baby may develop is likely to be attributable to events at delivery.
D. Tell them that your examination findings are reassuring, and that you will perform a careful developmental assessment at every well-child visit.
E. Avoid Speaking to the parents until you have had a chance to speak with the obstetrician and see the cord blood gas results.
D. The Apgar score at 1 minute reflects the neonatal environment immediately prior to birth and the 5-minute score correlates with the infant’s response to resuscitation. The Apgar scores are not an accurate reflection of morbidity. Your exami¬nation is a much better indicator of the child’s outcome, but you cannot rule out the possibility of cerebral palsy on· the basis of a normal neonatal physical examination. It is always best to leave discussion of the events of delivery to the obstetrician, keeping in mind that the majority of difficult deliveries are the result of a previously unidentified-antenatal insult. Avoidance of the parents at this emotional time will likely only further their anxiety and may impede your future efforts to provide for the child.
A term-appearing infant delivered virginally after an uneventful pregnancy develops vomiting and abdominal distention at 10 hours of life. No passage of stool has been recorded since birth. An abdominal radiograph shows distended bowel loops and a “bubbly” pattern in a portion of intestine, and the colon appears narrow. What do you tell the parents?
A. You would like to consult a geneticist because you suspect that their child has an unusual genetic defect that may in¬volve other associated anomalies.
B. The child most likely has necrotizing enterocolitis, a condi¬tion more commonly seen in premature infants. You there¬fore question the child’s supposed gestational age.
C. are concerned about the possibility of meconium ileus, and would like to obtain some family history.
D. You believe that the child is simply constipated and would like to change to a soy-based formula to see whether the baby tolerates this better. ‘
E. The child’s symptoms and radiograph findings are most likely normal.
C. Meconium ileus occurs when inspissated meconium ob¬structs the distal ileum. It is thought to be caused by deficiency of proteolytic enzymes. Obstruction begins in utero, resulting in underdevelopment of distal lumina. It is almost always associ¬ated with cystic fibrosis. Intestinal atresia and Hirschsprung (congenital aganglionic megacolon) cause clinical pictures sim¬ilar to this baby’s, but the radiographic findings for this child are most consistent with meconium ileus. Necrotizing enterocolitis also causes emesis and abdominal distension. but would be un¬likely in a term infant with no history of ischemic injury, and the colon would be expected to be of normal size. Constipation would not be consistent with the severity of this baby’s clinical picture, nor would it produce the described radiographic picture.
Appropriate clinical management of the patient in the above question includes:
A. Change from enteral feeds to intravenous fluids and obtain a genetics consult for the next morning.
B. Change from enteral feeds to intravenous fluids, obtain a blood culture, and initiate antibiotics.
C. Change from enteral feeds to intravenous fluids and obtain a STAT pediatric surgery consult.
D. Change from cow milk to soy-based infant formula and con¬tinue to observe the infant.
E. Do not change your current management.
C. Meconium ileus is a surgical emergency, as volvulus and per-foration peritonitis are not uncommon complications.
A term infant weighing 4 and a half kilogram is born without complication to a mother with class A gestational diabetes. His initial bedside glucose measurement is 30 mg/dL, but a subsequent glu¬cose level after the baby takes 30 cc of infant fornmla orally is 50 mg/dL, and a subsequent test 30 minutes later gives a glucose value of 55 mg/dL. His physical examination is unremarkable ex¬cept for his large size. On the proposed day of discharge 48 hours later, the baby appears mildly jaundiced. Vital signs are stable, and mother reports that he is eating well. Which laboratory tests are most likely to help you evaluate this infant’s jaundice?
A. Total and direct bilirubin and liver transaminases
B. Total and direct bilirubin, liver transaminases, and a hepatitis panel
C. Total bilirubin and a hematocrit
D. Total bilirubin and a complete blood count
E. Total and direct bilirubin and a complete blood count with differential and platelets
C. This baby most likely has hyperbilirubinemia secondary to liver immaturity, possibly complicated by polycythemia. You would expect him to have a high level of unconjugated bilirubin (included in the total bilirubin test), but his conjugated (or di¬rect) portion should be normal in the absence of intrahepatic disease.
A term infant is born to a mother with gestational dia¬betes. The mother had only intermittent prenatal care. The child requires endotracheal intubation at delivery for poor respiratory effort, tone and color. His initial serum glucose level is 10 mg/dL, which resolves over the first 36 hours of life with intra¬venous administration of dextrose solution. On the third day of life, his physical examination is remarkable for macrosomia and an abdominal mass. He passed meconium normally in the first 24 hours of life. The most likely cause of this abdominal mass is:
A. Liver engorgement
B. Infarction of the spleen
C. Small left colon syndrome
D. Hydronephrosis
E. Intraintestinal air
D. Renal vein thrombosis can present as an abdominal mass (hydronephrosis) in neonates after an episode of asphyxia. Such in-fants may have gross hematuria (or no hematuria at all), but mi-croscopic hematuria is more common. Hypertension is uncommon in the acute period following renal vein thrombosis but may occur as a late complication. The affected kidney may recover normal function. or it may atrophy. Bilateral renal vein thrombosis can lead to chronic renal failure. If the infant in this question had delayed passage of its first stool, small left colon syndrome, another complication of the infant of a diabetic mother, might also be considered as a cause of the abdominal mass.
An infant with Crigler-Najjar syndrome type I has developed bilirubin encephalopathy at 1 month of life. The hyperbiliru-binemia is caused by:
A. Increased production of bilirubin
B. Impaired conjugation of bilirubin
C. Deficient hepatic uptake of bilirubin
D. Severe deficiency of uridine diphosphate glucuronosyl- transferase
E. Glucose-6-phosphate dehydrogenase (G6PD) deficiency
D. Although all newborn infants are relatively deficient in the enzyme uridine diphosphate glucuronosyl transferase; those with Crigler-Najjar syndrome type 1 have as severe deficiency, leading to high bilirubin levels and encephalopathy :the mainstay of therapy is photoherapy, although research for other treatment options is going. Encephalopathy is rare in infants with Crigler-Najjar syndrome type II, in which serum biliribun levels rarely exceed 20 mg/dL.
A 2150-g infant is delivered at 34 weeks gestation. The mother had prenatal care in Mexico and states that she had no problems during her pregnancy. The mother’s highest temperature during labor was 100.8°F (38.2°C). The amniotic fluid had a brown stained appearance. The infant had a disseminated erythematous pustular rash at birth and had pallor, poor feeding, tachypnea, and cyanosis. The infant’s CBC reveals a marked monocytosis. The infant died at 4 hours of life soon after initiation of antibi-otics. This infant most likely had:
A. Disseminated herpes
B. Congenital syphilis
C. Listeriosis
D. Group B streptococcal disease
E. Congenital varicella
C. Listeria is a gram-positive rod isolated from soil, streams, sewage, certain foods, dust, and slaughterhouses. The food-borne transmission of disease is related to Mexican (soft ripened) cheese, whole and 2% milk, undercooked chicken and hot dogs, raw vegetables, and shellfish. The newborn infant ac¬quires the organism transparently or by aspiration or inges¬tion at the time of delivery. The mortality rate of early onset disease is approximately 30%.
A 2-hour-old term newborn male has coughing, choking, and cyanosis prior to feeding. A nasogastric tube is placed and meets resistance at 10 cm. Prenatal history is significant for polyhydramnios. Which of the following is most likely to be found in this infant?
A. Congenital cataracts
B. Microcephaly
C. Gingival hyperplasia
D. Hepatosplenomegaly
E. Vertebral anomaly
E. This infant is likely to have esophageal atresia. The VATER or VACTERL association of congenital anomalies includes vertetebral anomaly, anorectal malformation, tracheoesophageal fistula, cardiac (congenital heart disease), renal abnormalities, and limb defects.
A term male infant is born to a 33-year-old woman. She had little prenatal care, but denies having had any problems. Immedi-ately after birth the infant is noted to have cyanosis and respira-tory distress. Auscultation of the chest in the delivery room reveals right-sided heart sounds and absent breath sounds on the left. The most appropriate next step is to:
A. Perform a needle thoracostomy for possible pneumothorax.
B. Assess the abdomen to evaluate for possible congenital diaphragmatic hernia.
C. Order ultrasonography of the chest.
D. Prepare the infant for extracorporeal membranous oxygenation.
E. Order a computerized tomography of the chest.
B. Evaluation of the neonate born with respiratory distress with unilateral breath sounds must include examination of the ab¬domen. With asymmetric breath sounds both pneumothorax and congenital diaphragmatic hernia must be urgently considered. In this case, the infant was found to have a scaphoid abdomen increasing the likelihood that the infant has congenital dia¬phragmatic hernia; needle thoracostomy is contraindicated as intestinal perforation may occur. The patient must be stabilized immediately and the need for ECMO can only be ascertained af¬ter evaluating the infant’s response to initial therapy.
The next step in stabilizing the infant in the above question :
A. Perform endotracheal intubation and begin ventilatory support.
B. Begin bag-mask ventilation.
C. Perform a needle thoracostomy.
D. Call the ECMO team to initiate extracorporeal membranous oxygenation.
E. Administer intravenous sodium bicarbonate solution.
A. Endotracheal intubation with gentle ventilation should be initiated in an infant with congenital diaphragmatic hernia and respiratory distress. Bag-mask ventilation can lead to aeration of the thoracic intestinal contents leading to increased respira¬tory distress. Needle thoracostomy is contraindicated as intes¬tinal perforation may occur.
A 2-hour-old male infant presents with bilateral conjunctivitis. The next best step in management is to:
A. Administer prophylaxis with topical erythromycin.
B. Send the eye discharge for culture and start antimicrobial therapy based on culture results.
C. Start saline eye washes.
D. Begin systemic antibiotic therapy with erythromycin.
E. Begin systemic antibiotic therapy with ceftriaxone.
B. Conjunctivitis presenting in the first few days of life is most likely caused by chemical irritation. Laboratory evaluation of the eye discharge is performed; treatment usually can be with¬drawn and is based upon culture results.
A full-term newborn infant is having episodes of cyanosis and apnea, which are worse when he is attempting to feed, but he seems better when he is crying. Which of the following is the most important next step to quickly establish the diagnosis?
a. Echocardiogram
b. Ventilation perfusion scan.
c. Passage of catheter into nose .
d. Hemoglobin electrophoresis
e. Bronchoscopic evaluation of palate and larynx
The answer is c. It is important to make the diagnosis of choanal atresia quickly because it responds to treatment but can be lethal if unrecognized and untreated. Most neonates are obligate nose breathers and cannot breathe adequately through their mouths. Infants with choanal atresia have increased breathing difficulty during feeding and sleeping and improved respirations when crying. A variety of temporizing measures to maintain an open airway have been used, including oropharyngeal airways, position¬ing, tongue fixation, and endotracheal intubation, but surgical correction with placement of nasal tubes is most effective. The diagnosis can be made by failure to pass a catheter through the nose to the pharynx. Bronchoscopy would help diagnose lower airway anomalies. A ventilation-perfusion scan would be the appropriate examination if you were concerned about mismatch (e.g., that associated with pulmonary em¬bolism). A newborn with a hemoglobinopathy such as sickle cell or thalassemia would not present as this infant did, so an electrophoresis would not be helpful. An echocardiogram would be useful if you suspected con¬genital cyanotic heart disease. The lack of a murmur in a newborn does not rule out pathology: this would be a reasonable next step if the catheter passed through both nares without difficulty.
A full-term newborn infant is having episodes of cyanosis and apnea, which are worse when he is attempting to feed, but he seems better when he is crying. Which of the following is the most important next step to quickly establish the diagnosis?
a. Echocardiogram
b. Ventilation perfusion scan.
c. Passage of catheter into nose .
d. Hemoglobin electrophoresis
e. Bronchoscopic evaluation of palate and larynx
The answer is c. It is important to make the diagnosis of choanal atresia quickly because it responds to treatment but can be lethal if unrecognized and untreated. Most neonates are obligate nose breathers and cannot breathe adequately through their mouths. Infants with choanal atresia have increased breathing difficulty during feeding and sleeping and improved respirations when crying. A variety of temporizing measures to maintain an open airway have been used, including oropharyngeal airways, position¬ing, tongue fixation, and endotracheal intubation, but surgical correction with placement of nasal tubes is most effective. The diagnosis can be made by failure to pass a catheter through the nose to the pharynx. Bronchoscopy would help diagnose lower airway anomalies. A ventilation-perfusion scan would be the appropriate examination if you were concerned about mismatch (e.g., that associated with pulmonary em¬bolism). A newborn with a hemoglobinopathy such as sickle cell or thalassemia would not present as this infant did, so an electrophoresis would not be helpful. An echocardiogram would be useful if you suspected con¬genital cyanotic heart disease. The lack of a murmur in a newborn does not rule out pathology: this would be a reasonable next step if the catheter passed through both nares without difficulty.
A mother in the newborn nursery wishes to breast-feed her newborn infant, but is worried about medical conditions that would prohibit her from doing so. You counsel her that of her listed conditions, which of the following is a contraindication to breast-feeding?
a. Upper respiratory tract infection
b. Cracked and bleeding nipples
c. Mastitis
d. Inverted nipples
e. HIV infection
The answer is e. There are few contraindications to breast-feeding. Active pul¬monary tuberculosis and HIV are two examples of infectious contraindica¬tions in developed countries, as well as malaria, typhoid fever, and septicemia. All medications taken by the mother will be secreted in breast milk, but usually not in amounts significant enough to affect the infant. Mothers taking antineoplastic agents should not breast-feed. Mothers with mastitis can continue to breast-feed; frequent feedings may actually help the condition by preventing engorgement. Mothers with mild viral illness may also continue to breast-feed. Cracked or bleeding nipples may make breast-feeding uncomfortable, but are not contraindications. Inverted nip¬ples can usually be remedied, and only rarely prohibit breast-feeding.
A mother delivers a neonate with meconium staining and Apgar score of 3 at 1 and 5 mm of life. She had no prenatal care and the delivery was by emergency cesarean section for severe fetal bradycardia. Which of the following sequelae could be expected to develop in this intubated neo¬nate with respiratory distress?
a. Sustained rise in pulmonary arterial pressure
b. Hyperactive bowel sounds
c. Microcephaly with micrognathia
d. Cataracts
e. Thrombocytosis
The answer is a. The low Apgar scores, meconium stain¬ing, and ensuing respiratory distress suggest that asphyxia has occurred. During a period of asphyxia, the resulting hypoxemia, acidosis, and poor perfusion can damage a neonate’s brain, heart, kidney, liver, and lungs. The resulting clinical abnormalities include cerebral edema, irritability, sei¬zures, cardiomegaly, heart failure, renal failure, poor liver function, dis¬seminated intravascular coagulopathy, and respiratory distress syndrome. There can be excessively high pulmonary arterial pressure at the same time systemic blood pressure begins to fall, resulting in a persistent right-to-left shunt across a patent ductus arteriosus or foramen ovale.
A 2-year-old boy is being followed for congenital cytomegalovirus (CMV) infection. He is deaf and developmentally delayed. The child’s mother informs you that she has just become pregnant and is concerned that the new baby will be infected. Which of the following is true? .
a. The mother has antibodies to CMV that are passed to the fetus
b. The mothers infection cannot become reactivated
c. The likelihood that the new baby will become clinically ill is approximately 1%
d. Termination of pregnancy is advised
e. The new infant should be isolated from the older child .
The answer is a. Cytomegalovirus infection is the most common cause of congenital infection, occurring in 0.2 to 2.4% of all live births. Cytomegalic inclusion disease is a constellation of findings including hepatomegaly, splenomegaly, jaundice, petechiae, purpura, and micro¬cephaly. In the United States, 20 to 90% of women of childbearing age have serologic evidence of a past infection with CMV. Symptomatic congenital disease usually occurs when a mother has a primary CMV infection in the first trimester of pregnancy. Many of these babies die. and those who survive are severely affected. In the event of reactivation of CMV infection during pregnancy, maternal IgG, passed transplacentally, protects the infant from serious infection. Although most infants infected during this sec¬ondary maternal infection are asymptomatic, about l% of them eventu¬ally manifest hearing and neurologic problems. Some recommend keeping a child with congenital CMV infection away from susceptible pregnant (or about to become pregnant) women because CMV excretion can persist for months to years; at the very least, good hand-washing practices should be instituted. If infected shortly after birth, the younger sibling will probably be asymptomatic since he or she has maternal IgG in the circulation. CMV is primarily an occult infection. Of toddlers in day-care centers, 20 to 80% acquire CMV and shed it in saliva and urine for years. Diagnosis is made with isolation of the virus from urine, saliva, or other secretions.
A full-term infant is born after a normal pregnancy; delivery, however complicated by marginal placental separation. At 12 h of age, the child, although appearing to be in good health, passes a bloody meconium stool. For determining the cause of the bleeding, which of the following diagnostic procedures should be performed first?
a. A barium enema
b. An Apt test
c. Gastric lavage with normal saline
d. An upper gastrointestinal series
e. A platelet count, prothrombin time, and partial thromboplastin time
The answer is b. Hematemesis and melena are not uncommon in the neonatal period, espe¬cially if gross placental bleeding has occurred at the time of delivery. The diagnostic procedure that should be done first is the Apt test, which differ¬entiates fetal from adult hemoglobin in a bloody specimen. The test is based upon the finding that fetal hemoglobin is alkali resistant, while adult hemoglobin will convert to hematin upon exposure to alkali. If the blood in an affected infants gastric contents or stool is maternal in origin, further workup of the infant is not needed.
As you are about to step out of a newly delivered mothers room, she mentions that she wants to breast-feed her healthy infant, but that her obstetrician was concerned about one of the medicines she was taking. Which of the woman’s medicines, listed below, is clearly contraindicated in breast-feeding?
a. Ibuprofen as needed for pain or fever
b. labetalol for her chronic hypertension
c. Lithium for her bipolar disorder
d. Carbamazepine for her seizure disorder
e. Acyclovir for her HSV outbreak
The answer is c. Most medica¬tions are secreted to some extent in breast milk, and some lipid-soluble medications may be concentrated in breast milk. Although the list of contraindicated medications is short, caution should always be exercised when giving a medication to a breast-feeding woman. Medications that are clearly contraindicated include lithium, cyclosporin, anteneoplastic agents, illicit drugs including cocaine and heroin, ergotamines, and bromocreptine (which suppresses lactation). Although some suggest that oral contracep¬tives may have a negative impact on milk production, the association has not been proven conclusively. In general, antibiotics are safe, with only a few exceptions (such as tetracycline). While sedatives and narcotic pain medications are probably safe, the infant must be observed carefully for sedation. All of the medications listed in the question are considered safe, except for lithium.
The nurse from the level 2 neonatal intensive care nursery calls you to evaluate a baby. The infant, born at 32 weeks’ gestation, is now 1 week old and had been doing well on increasing nasogastric feedings. This after¬noon , however, the nurse noted that the infant has vomited the last two feedings and seems less active. Your examination reveals a tense and dis-tended abdomen with decreased bowel sounds. As you are evaluating the child, he has a grossly bloody stool. Your initial management of this infant should include which of the following?
a. Surgical consultation for an emergent exploratory laparotomy
b. Continued feeding of the infant, as gastroenteritis is usually self-limited
c. Stool culture to identify the etiology of the bloody diarrhea and an infectious diseases consultation
d. Stopping feeds, beginning intravenous fluids, ordering serial abdominal films and initiating systemic antibiotics
e. Upper GI series and barium enema to evaluate for obstruction
The answer is d. The infant presented in the question has necrotizing enterocolitis. NEC, a potentially life-threatening disease of the neonate. It is more common in premature infants, but has been described in term infants as well. Although several organisms have been isolated from NEC patients, no clear cause for this condition has been identified. Patients present with feeding intolerance and a distended abdomen, about a quar¬ter have grossly bloody stool. Pneumatosis intestinalis is found on plain radiograph of the abdomen and is diagnostic for NEC in this age group. Management depends initially on the presence or absence of perforation; it no evidence of free peritoneal air is found, the infant should be put on bowel rest with nasogastric decompression, and systemic antibiotics are initiated. Electrolytes and vital signs should be monitored closely, and se¬rial abdominal films should be performed to evaluate for perforation. If free air is identified on plain radiographs or if the infant clinically worsens with medical management, surgical consultation is required. An exploratory laparotomy is usually performed, and any necrotic intestinal tissue is removed. Occasionally, removal of necrotic gut will result in an infant with¬out adequate intestinal surface area to absorb nutrition, a condition known as short bowel syndrome.
recovering premature infant who weighs 950 g is fed breast milk to provide 120 cal/kg/d. Over the ensuing weeks, the baby is most apt to develop which of the following?
a. Hypernatremia
b. Hypocalcemia
c. Blood in the stool
d. Hyperphosphatemia
e. Vitamin D toxicity
.
The answer is b. It usually is impossible with any combination of parenteral and enteral nutrition to match what the infant would have accumulated in utero. The average healthy, low-birth-weight infant of this size requires a daily intake of cal¬cium of about 200 mg/kg. Breast milk has much less calcium (and phos¬phorus) than do commercial formulas. The breast milk can be directly supplemented with calcium, or it can be supplemented with commercial fortifiers. Alternatively, it can be mixed with formulas designed for the premature infant. Breast milk promotes gut maturation and prevents intestinal atrophy induced by lack of enteral feeding; however, it is likely to have insufficient calcium and phosphorus for catch-up growth
An infant weighing 1400 g (3 Ib) is born at 32 weeks’ gestation. Initial evaluation was benign, and the infant was transferred to the level 2 nursery for prematurity. The nurse there calls at an hour of life and reports the infant is tachypneic. Vital signs include a heart rate of 140, a respiratory rate of 80/min, a temperature of 35°C (95°F), and a peripheral oxygen sat¬uration of 98%. The lungs are clear with bilateral breath sounds and there is no murmur; the infant is in no distress. Which of the following is the most appropriate first step in evaluating the infant?
a. Obtain a complete blood count and differential.
b. Perform a lumbar puncture
c. Obtain a chest radiograph
d. Place the infant under a warmer
e. Administer supplemental oxygen
The answer is d. A room temperature of 24°C provides a cold environment for newborn infants. Aside from the fact that these infants emerge from a warm, 37.6°C (99.5°F) intrauterine environment, at birth, infants (and especially preterm infants) are wet, have a relatively large surface area for their weight, and have little subcutaneous fat. Within minutes of delivery, the infants are likely to become pale or blue and their body temperature will drop. In order to bring body temperature back to normal, they must increase their metabolic rate; ventilation, in turn, must increase propor¬tionally to ensure an adequate oxygen supply. Because a preterm infant is likely to have respiratory problems and be unable to oxygenate adequately, lactate can accumulate and lead to a metabolic acidosis. Infants rarely shiver in response to a need to increase heat production. If the tachypnea persists after warming the infant, sepsis, pneumonia, and primary surfac-tant deficiency are all possible; thus, several of the alternative answers may then be appropriate.
primiparous woman whose blood type is O positive gives birth at term to an infant who has A-positive blood and a hematocrit of 55%. A serum bilirubin level obtained at 36 h of age is 12 mg/dL. Which of the fol¬lowing laboratory findings would be characteristic of ABO hemolytic dis¬ease?
a. A normal reticulocyte count
b. A positive direct Coombs test
c. Crescent-shaped red blood cells in the blood smear
d. Elevated hemoglobin
e. Petechiae
The answer is b. If a mother is positive and her baby is A positive, the baby has a chance of developing hemolytic disease. Hemolytic disease and jaun¬dice caused by a major blood-group incompatibility are usually less severe than with Rh incompatibility. Although the hematocrit of affected infants usually is normal, elevation of the reticulocyte count and the presence of nucleated red blood cells and micro spherocytes in the blood smear provide evidence of hemolysis. In comparison with hemolytic disease caused by Rh incompatibility, where it is usually strongly positive, major blood-group incompatibility is often associated with a direct Coombs test that is fre¬quently weakly positive. Petechiae are usually associated with decreased number of platelets. Crescent-shaped (sickled) red blood cells are found with sickle cell disease.
Two infants are born at 36 weeks’ gestation. Infant A weighs 2600 g and infant B weighs 1600 g. Infant B is more likely to have which of the following conditions?
a. Congenital malformations
b. Low hematocrit
c. Hyperglycemia
d. Surfactant deficiency
e. Rapid catch-up growth
The answer is a. Small-for-dates infants are subject to a differ¬ent set of complications than preterm infants whose size is appropriate for gestational age. The small-for-dates infants have a higher incidence of major congenital anomalies and are at increased risk for future growth retardation, especially if length and head circumference as well as weight are small for gestational age. Also more common are neonatal asphyxia and the meconium aspiration syndrome, which can lead to pneumothorax, pneumomediastinum. or pulmonary hemorrhage. These, rather than hya¬line membrane disease, are the major pulmonary problems in SGA infants. Because neonatal symptomatic hypoglycemia is more commonly found in small-for-dates infants, careful blood glucose monitoring and early feeding are appropriate precautions. Normal or elevated hematocrit is also more common in these infants.
3-day-old infant, born at 32 weeks’ gestation and weighing 1700 g, has three episodes of apnea, each lasting 20 to 25 s and occur-ring after a feeding. During these episodes, the heart rate drops from 140 to 100 beats per min, and the child remains motionless; between episodes, however, the child displays normal activity. Blood sugar is 50 mg/dL and serum calcium is normal. Which of the following is most likely true regard¬ing the child’s apneic periods?
a. They are due to an immature respiratory center
b. They are a part of periodic breathing
c. They are secondary to hypoglycemia
d. They are manifestations of seizures
e. They are evidence of underlying pulmonary disease
The answer is a. Apneic episodes are characterized by an absence of respi¬rations for more than 20 s and may be accompanied by bradycardia and cyanosis. A large number of conditions can cause apnea. Periods of apnea are generally thought to be secondary to an incompletely developed respi¬ratory center, particularly when they are seen, as is common, associated with prematurity. Although seizures, hypoglycemia, and pulmonary dis¬ease accompanied by hypoxia can lead to apnea, these causes are less likely in the infant described, given that no unusual movements occur during the apneic spells, that the blood sugar level is more than 40 mg/dL, and that the child appears well between episodes. Other, less common, explanations for central apnea include congenital central hypoventilation syndrome (formerly known as Ondine’s curse), Arnold-Chiari malformations, and congenital infections. Periodic breathing, a common pattern of respi¬ration in low-birth-weight babies, is characterized by recurrent breathing pauses of 3 to 10 s.
You have a 2-week-old term infant in your office for a well-child visit. The mother notes that she received a letter from the states Department of Health reporting that her child’s newborn screen had come back abnormal, indicating possible congenital hypothyroidism. Your nursing staff has already repeated the laboratory studies, and the child’s TSH is 32 (elevated) and the T4 is 0.3ug/dL (low). Which of the following is the most appropriate management at this point?
a. Begin sodium-L-thyroxin, 10 to 15 ug/kg/day
b. Serum for maternal antithyroid antibodies
c. Referral to endocrinology for evaluation
d. A-sodium iodine uptake scan
e. Ultrasound of the thyroid gland
The answer is a. most developed countries, screen in the neonatal period infants for congenital hypothyroidism. Identified and treated early, most affected children will have normal growth and intelligence. Untreated, the outcome of congeni¬tal hypothyroidism is mental retardation and poor growth (cretinism). Early treatment is essential and should not be delayed while arranging fur¬ther diagnostic evaluation. Approximately 85% of all cases of congenital hypothyroidism are a result of thyroid dysgenesis, generally a sporadic event. A sodium iodine uptake scan can help identify the underlying cause, but treatment should not be withheld pending the outcome of this or any other study after initial confirmation.
The father of a 1-week-old infant comes to the office in a panic. He has just noticed on his child a right anterior shoulder mass that seems tender. The father is an osteosarcoma survivor and fears the child has the same malig¬nancy. In reviewing the baby’s discharge papers, you note the child was a term, appropriate-for-gestational-age vaginal delivery with a birth weight of 3200 g. Apgar scores were 9 at 1 and 5 min. Your examination is significant for a large firm mass on the right clavicle; the rest of the examination is normal. Management of this problem should include which of the following?
a. Magnetic resonance imaging of the right shoulder
b. Reassurance and supportive care
c. A biopsy of the mass for culture and cytology
d. Referral to an orthopedic surgeon
e. Skin biopsy to test for osteogenesis imperfecta
The answer is b. The clavicle is the most commonly fractured bone in the delivery process. While some fractures are identified at birth, others may not be identified until callus formation is noted at about a week of age. Clavicular fracture may happen in any delivery, although there is higher risk with large-for-gestational-age infants. Initial presentation of a fractured clavicle may include a pseudoparalysis, in which the infant refuses to move the ipsilateral arm, mimicking an Erb-Duchenne paralysis.
A 1-day-old infant who was born by a difficult forceps delivery is alert and active. She does not move her left arm, however, which she keeps it internally rotated by her side with the forearm extended and pronated: she also does not move it during a Moro reflex. The rest of her physical examination is normal. This clinical picture most likely indicates which of the following?
a. Fracture of the left clavicle
b. Fracture of the left humerus
c. Left-sided Erb-Duchenne paralysis
d. Left-sided Klumpke paralysis
e. Spinal injury with left hemiparesis
The answer is c. In a difficult delivery in which traction is applied to the head and neck, several injuries, including all those listed in the question, may occur. Erb-Duchenne paralysis affects the fifth and sixth cervical nerves; the affected arm cannot be abducted or externally rotated at the shoulder, and the forearm cannot be supinated. Injury to the seventh and eighth cervical and first thoracic nerves (Klumpke paralysis) results in palsy of the hand and also can produce Horners syndrome. Fractures in the upper limb are not associated with a characteristic posture, and passive movement usually elicits pain. Spinal injury causes complete paralysis below the level of injury. When paralysis of an upper extremity from injury to the brachial plexus is found in a neonate, injury to the phrenic nerve should also be suspected because the nerve roots are close together and can be injured concurrently. The paralyzed diaphragm can be noted to remain elevated on a chest x-ray taken during deep inspiration when it will con¬trast with the opposite normal diaphragm in its lower normal position; on expiration, this asymmetry cannot be seen. On inspiration, not only is breathing impaired since the paralyzed diaphragm does not contract, but also the negative pressure generated by the intact diaphragm pulls the mediastinum toward the normal side, impairing ventilation further. The diagnosis can easily be made by fluoroscopy, where these characteristic movements on inspiration and expiration can be seen. Rarely, both diaphragms can be paralyzed, producing much more severe ventilatory impairment. Fortunately, these injuries frequently improve spontaneously.
You are examining a newborn infant in the well-baby nursery. The infant was the product of a benign pregnancy and vaginal delivery. Interestingly, your measurement of fronto-occipital head circumference is about 2 cm larger than the initial measurement done several hours before. Your examination otherwise is significant for tachycardia and a “‘squishy” feel i.e the entire scalp. You can elicit a fluid wave over the scalp. Management of this condition should include which of the following?
a. Transfer to the newborn ICU
b. Observation and parental reassurance
c. CT scan of the skull with bone windows
d. Surgical drainage
e. Elevation of the head of the crib
The answer is a. A subgaleal or subaponeurotic hemorrhage can be life threat¬ening; infants may lose a third or more of their blood volume into this potential space, leading to hypovolemic shock. The other two more com¬mon findings on a newborns head, caput succedaneum and cephalohematoma, may be distinguished by examination. Caput is edema and will feel firm and nonfluctuant; it may cross the midline. Cephalohematoma is limited by its location under the periosteum of the individual bones of the skull, so the lesion stops at suture lines and does not cross the midline. Cephalohematomas will typically feel fluid-filled or fluctuant. A subgaleal hemorrhage will feel like a cephalohematoma that crosses the midline. In a situation as described in the question, fluid resuscitation may be required, as may be transfusion. Careful monitoring is essential. Trying to identify the source of bleeding is rarely successful, so a CT is of limited benefit. Observation alone in this case could lead to shock and death. Surgical drainage is contraindicated except in cases of infected hematoma.
A 19-year-old primiparous woman develops toxemia in her last trimester of pregnancy and during the course of her labor is treated with magnesium sulfate. At 38 weeks’ gestation, she delivers a 2100-g infant with Apgar scores of 1 at 1 min and 5 at 5 min. Laboratory studies at 18 h of age reveal a hematocrit of 79%, platelet count of 100,000/uL, glucose 41 mg/dL, magnesium 2.5 mEq/L, and calcium 8.7 mg/dL Soon after, the infant has-a generalized convulsion. Which of the following is the most likely cause of the infant’s seizure?
a. Polycythemia
b. Hypoglycemia
c. Hypocalcemia
d. Hypermagnesemia
e. Thrombocytopenia
The answer is a. An infant of 2100 g at 38 weeks would be considered small for gestational age (SGA), a not uncommon consequence of maternal toxemia. Pregnancy induced hypertension can produce a decrease in uteroplacental blood flow and areas of placental infarction. This can result in fetal nutritional deprivation and intermittent fetal hypoxemia, with a decrease in glycogen storage and a relative erythrocytosis, respec-tively. Hence, neonatal hypoglycemia and polycythemia are common clini¬cal findings in these infants. A blood glucose level of 30 mg/dL in a full-term infant, however, is probably normal during the first postnatal day, and an infant is very unlikely to have a convulsion as a result of a level of 41 mg. Serum calcium levels usually decline during the first 2 to 3 postna¬tal days, but will only be considered abnormally low in a term infant when they fall below 7.5 to 8 mg/dL. Neonatal hypermagnesemia is common in an infant whose mother has received MgSO4 therapy, but is usually asymp¬tomatic or produces decreased muscle tone or floppiness. A persistent venous hematocrit of greater than 65% in a neonate is regarded as polycythemia and will be accompanied by an increase in blood viscosity.
Manifestations of the “hyperviscosity syndrome” include tremulousness or jitteriness that can progress to seizure activity because of sludging of blood in the cerebral microcirculation or frank thrombus formation, renal vein thrombosis, necrotizing enterocolitis, and tachypnea. Therapy by partial exchange transfusion with saline or lactated Ringers solution is preferred, and may be more likely to be useful if per¬formed prophylactically before significant symptoms have developed, but literature looking at outcomes in these infants is lacking.
After an uneventful labor and delivery, an infant is born at 32 weeks’ gestation weighing 1500 g. Respiratory difficulty develops immediately after birth and increases in intensity thereafter. At 6 h of age, the child’s respiratory rate is 60 breaths per min. Examination reveals grunting, intercostal retraction, nasal flaring, and marked cyanosis in room air. Auscultation reveals poor air movement. Physiologic abnormalities compatible with these data include which of the following?
a. Deceased lung compliance, reduced lung volume, left-to-right shunt of blood
b. Decreased lung compliance, reduced lung volume, right-to-left shunt of blood
c. Decreased lung compliance, increased lung volume, left-to-right shunt of blood
d. Normal lung compliance, reduced lung volume, left-to-right shunt of blood
e. Normal lung compliance, increased lung volume, right-to-left shunt of blood
The answer is b. For the child described in the question, prematurity; and the clinical picture presented make the diagnosis of hyaline membrane disease (infant respiratory distress syndrome) likely. HMD is caused by surfactant deficiency, and the incidence is increased with decreasing gestational age and birth weight. In this disease, lung compliance is reduced; lung volume is also reduced, and a significant right-to-left shunt of blood, can occur. Some of the shunt can result from a patent ductus arteriosus or foramen ovale, and some can be due to shunting in the lung. Minute ventilation is higher than normal, and affected infants must work harder in order to sustain adequate respiration.
A term infant is born to a known HIV-positive mother. She has been taking antiretroviral medications for the weeks prior to the delivery of her infant. Routine management of the healthy infant should include which of the, following?
a. Admission to the neonatal intensive care unit for close cardiovascular monitoring
b. HIV EL1SA on the infant to determine if congenital infection has occurred
c. A course of zidovudine for the infant
d. Chest radiographs to evaluate for congenital Pneumocystis carinii
e. Administration of IV1G to the baby to decrease the risk of perinatal HIV infection
The answer is c. The transmission of HIV from mother to infant has decreased in recent years, due in large part to perinatal administration of antiretroviral medications to the mother and a course of zidovudine to the exposed infant. Studies suggest that a better than 50% decrease in transmission can be seen with appropriate medications as outlined.
An ELISA is an antibody test and will be positive in the infant born to an HIV-infected mother due to maternal antibodies that are passed through the placenta; it is not a useful test in the newborn infant to determine neonatal infection. IVIG has not been shown to have a role in decreasing perinatal transmission. Healthy asymptomatic term infants born to HIV-infected mothers do not need special monitoring, nor do they need routine radiographs.
Initial examination of a full-term infant weighing less than 2500 g shows edema over the dorsum of her hands and feet. Which of the following findings would support a diagnosis of Turner syndrome?
a. A liver palpable to 2 cm below the costal margin
b. Tremulous movements and ankle clonus
c. Redundant skin folds at the nape of the neck
d. A transient, longitudinal division of the body into a red half and a pale half
e. Softness of the parietal bones at the vertex
The answer is c. Turner syndrome is a genetic disorder; the 45, X karyotype is the most common. At birth, affected infants have low weights, short stature, edema over the dorsum of the hands and feet, and loose skin folds at the nape of the neck. Some other findings with this syndrome include sexual infantilism, streak gonads, typical faces, shield chest, low hairline, coarctation of the aorta, hypertension, bicuspid aortic valve, high palate, and horseshoe kidney. Coarse, tremulous movements accompanied by ankle clonus; vascular instability as evidenced, for example, by a harlequin color change (a transient, longitudinal division of a body into red and pale halves); softness of parietal bones at the vertex (craniotabes); and a liver that is palpable down to 2 cm below the costal margin are all findings often demonstrated by normal infants and are of no diagnostic significance in the clinical situation presented.
Shortly after birth, an infant develops abdominal distention and begins to drool. When she is given her first feeding, it runs out the side of her mouth, and she coughs and chokes. Physical examination reveals tachypnea, intercostal retractions, and bilateral pulmonary rales. The esophageal anomaly that most commonly causes these signs and symptoms is illustrated by which of the following?
a. Figure A
b. Figure B
c. Figure C
d. Figure D
e. Figure E
The answer is d. Abdominal distention, choking, drooling, and coughing associated with feedings are symptoms of esophageal anomalies. The anomaly illustrated by D is the most common: that A can be diag¬nosed after repeated episodes of pneumonia. The anomalies in E and C are associated with all the same symptoms except abdominal distention, which cannot develop because air cannot enter the gastrointestinal tract. B and C are the least common; in these, the upper esophageal segment is connected directly to the trachea, and massive entry of fluid into the lungs occurs.
VATER association, a complex of cardiovascular malformations, skele¬tal malformations, and renal abnormalities, has tracheoesophageal fistula as a common finding.
You are speaking to expectant parents who are expecting their first baby in about 2 weeks. They are concerned about the safety of childhood immu¬nizations and also about “unneeded” medications given to newborns in the hospital. They ask about the purpose of the routine administration of intra¬muscular vitamin K. You explain to them about hemorrhagic disease of the newborn, stating that the untreated baby can manifest which of the following?
a. deficiency of factor V
b. A prolonged prothrombin time and a risk of serious hemorrhage in the days following delivery
c. Development of hemorrhagic manifestations within 24 h of delivery
d. A prolonged bleeding time and decreased fibrinogen level
e. Hemorrhagic symptoms at a much higher rate if the infant is fed cow’s milk rather than breast milk
The answer is b. Failure to give vitamin K prophylactically to newborn infants is associated with a decline in the levels of vitamin K-dependent coagulation factors. In less than 1% of infants (but especially those fed human breast milk), the levels reached are low enough to pro¬duce classic hemorrhagic manifestations on the second to seventh day of life. These manifestations include melena, hematuria, and bleeding from the circumcision; intracranial hemorrhage and hypovolemic shock are seri¬ous complications. Diagnosis of this condition is indicated by a prolonged prothrombin time, which reflects inadequate concentrations of factors II, VII, IX, and X. Bleeding time, fibrinogen, and platelets are normal.
You are advised by the obstetrician that the mother of a baby she has delivered is a carrier of hepatitis B surface antigen (HBsAg-positive). Which of the following is the most appropriate action in managing this infant?
a. Screen the infant for HBsAg
b. Isolate the infant with enteric precautions
c. Screen the mother for hepatitis B “e” antigen (HBeAg)
d. Administer hepatitis B immune globulin and hepatitis B vaccine to the infant
e. Do nothing because transplacentally acquired antibody will prevent infection in the infant
The answer is d. The infant of a mother who is a carrier of hepatitis B surface antigen has a significant risk of acquiring infection. This usually occurs at the time of delivery, but infection can also be acquired dur¬ing pregnancy and postnatally. A small percentage of infected neonates develop acute icteric hepatitis, but the majority remain asymptomatic. Of these infected asymptomatic infants, 80% or more will develop chronic infection, the long-term consequences of which are chronic liver disease and, possibly, hepatocellular carcinoma. Combined passive-active immunoprophylaxis in the form of hepatitis B immune globulin and hepatitis B vac¬cine affords protection not only from immediate perinatal infection but also from infection that may be acquired as a result of continued exposure in the household of a chronic carrier.
Immunization in this infant is indicated regardless of the presence of hepatitis B “e” antigen (HBeAg) in the mother. Although the presence of HBeAg, especially in the absence of antibody to HBeAg, is associated with high rates of transmission to neonates, any woman positive for hepatitis B surface antigen (HBsAg) is potentially infectious. It is not necessary to iso¬late infants born to carriers of HBsAg, and screening of neonates for HBsAg is not indicated. Testing for HBsAg and anti-HBsAg at least 1 month after the third dose of hepatitis B vaccine will determine the efficacy of these measures.
You are called to a delivery of a term infant, about to be born via cesarean section to a mother with multiple medical problems, including a 1-month history of a seizure disorder, for which she takes phenytoin; rheumatic heart disease, for which she must take penicillin daily for life; hypertension, for which she takes propranolol; acid reflux, for which she takes aluminum hydroxide; and a deep venous thrombosis in her left calf diagnosed two days ago, for which she was started on a heparin infusion. The obstetrician is concerned about the possible effects of the mothers multiple medications on the newborn infant. Which of the following med¬ications is most likely to cause harm in this newborn infant at delivery?
a. Propranolol
b. Penicillin
c. Aluminum hydroxide
d. Phenytoin
e. Heparin
The answer is a. The effect of a drug on the fetus is determined by the nature of the drug and by the timing and degree of exposure. Heparin does not cross the placental barrier and does not appear to directly affect the fetus once pregnancy is well established. Phenytoin may cause birth defects when given during the first trimester. Penicillin and aluminum hydroxide have not been found to affect the fetus. Propranolol, which may cause growth retardation when given throughout pregnancy, diminishes the ability of an asphyxiated infant to increase heart rate and cardiac output. It has also been associated with hypoglycemia and apnea.
The mother and father of a newborn come in for the 2-week checkup. The mother complains of “colic” and asks if she can switch to goats milk instead of breast milk. Which of the following should be your main concern about using goat’s milk instead of breast milk or cow’s milk?
a. It has insufficient calories
b. It has insufficient folate
c. It has insufficient whey
d. It has insufficient casein
e. It has insufficient fat
The answer is b. Goat’s milk, by itself, is not an ideal source of infant nutrition, as it contains inadequate folate and iron, potentially contributing to anemia. Caloric content is actually more dense than cow’s milk-based formulas (about 30 kcal/oz, compared to 20 kcal/oz for for¬mula and breast milk). Casein and whey are the protein sources in goat’s milk, as they are for many formulas. Some manufacturers in different coun¬tries produce a goat milk-based formula that contains supplemental vita¬mins and iron. Unpasteurized goat’s milk should never be used as an infant formula because of the risk of brucellosis.
A family friend has a 2-day-old infant. She is concerned because the pediatrician noted the child was “yellow” and ordered some studies. For her own reassurance, she and her husband ask you to review the laboratory studies. Both the mother and baby have O-positive blood. The baby’s di¬rect serum bilirubin is 0.2 mg/dL, with a total serum bilirubin of 11.8 mg/dL. The infant’s white count is 13,000/|J.L with a differential of 50% polymorphonuclear cells, 45% lymphocytes, and 5% monocytes. The hemoglobin is 17 g/dL, and the platelet count is 278,000/|iL. Reticulocyte count is 1.5%. The peripheral smear does not show fragments or abnormal cell shapes. Which of the following is the most likely explanation for this infant’s skin color?
a. Rh or ABO hemolytic disease
b. Physiologic jaundice
c. Sepsis
d. Congenital spherocytic anemia
e. Biliary atresia
The answer is b. The development of jaundice in a healthy full-term baby may be considered the result of a normal physiologic process if the time of onset and duration of the jaundice and the pattern of serially deter¬mined serum concentrations of bilirubin are in conformity with currently-accepted safe criteria. Physiologic jaundice becomes apparent on the sec¬ond or third day of life, peaks to levels no higher than about 12 mg/dL on the fourth or fifth day and disappears by the end of the week. The rate of rise is less than 5 mg/dL per 24 h and levels of conjugated bilirubin do not exceed about 1 mg/dL. Concern about neonatal jaundice relates to the risk of the neurotoxic effects of unconjugated bilirubin.
The precise level and duration of exposure necessary to produce toxic effects are not known, but bilirubin encephalopathy, or kernicterus, is rare in term infants whose bilirubin level is kept below 18 to 20 mg/dL. Certain risk factors affecting premature or sick newborns increase their susceptibil¬ity to kernicterus at much lower levels of bilirubin. The diagnosis of phys¬iologic jaundice is made by excluding other causes of hyperbilirubinemia by means of history, physical examination, and laboratory determinations.
Jaundice appearing in the first 24 h is usually a feature of hemolytic states and is accompanied by an indirect hyperbilirubinemia, reticulocytosis, and evidence of red-cell destruction on smear. In the absence of blood group or Rh incompatibility, congenital hemolytic states (e.g., spherocytic anemia) or G6PD deficiency should be considered. With infection, hemo¬lytic and hepatotoxic factors are reflected in the increased levels of both direct and indirect bilirubin. Studies should include maternal and infant Rh types and blood groups and Coombs tests to detect blood-group or Rh incompatibility and sensitization.
Measurements of total and direct bilirubin concentrations help to determine the level of production of bilirubin and the presence of conju¬gated hyperbilirubinemia. Hematocrit and reticulocyte count provide in¬formation about the degree of hemolysis and anemia, and a complete blood count screens for the possibility of sepsis and the need for cultures.
examination of the blood smear is useful in differentiating common hemolytic disorders. Except for determinations of total and direct biliru¬bin, tests of liver function are not particularly helpful in establishing the cause of early-onset jaundice. Transient elevations of transaminases (AST and ALT) related to the trauma of delivery and to hypoxia have been noted.
Biliary atresia and neonatal hepatitis can be accompanied by elevated levels of transaminase but characteristically present as chronic cholestatic jaundice with mixed hyperbilirubinemia after the first week of life.
The mother of a 2-week-old infant reports that since birth, her infant sleeps most of the day; she has to awaken her every 4 h to feed, and she will take only an ounce of formula at a time. She also is concerned that the infant has persistently hard, pellet-like stools. On your examination you find an infant with normal weight and length, but with an enlarged head. The heart rate is 75 bpm and the temperature is 35°C (95°F). The child is still jaundiced. You note large anterior and posterior fontanelles. a dis¬tended abdomen, and an umbilical hernia. This clinical presentation is likely a result of which of the following?
a. Congenital hypothyroidism
b. Congenital megacolon (Hirschsprung disease)
c. Sepsis
d. Infantile botulism
e. Normal development
The answer is a. The clinical findings of congenital hypothyroidism are subtle, and may not be present at all at birth; this is thought to the result of passage of some T4 transplacentally. Infants with examination findings will usually have an umbilical hernia and a distended abdomen. The head may be large, and the fontanelles will be large as well. The child may be hypothermic and have feeding difficulties; constipation and jaun¬dice may be persistent. Skin may be cold and mottled, and edema may be found in the genitals and extremities. The heart rate may be slow, and ane¬mia may develop. As these findings may be subtle or nonexistent, neonatal screening programs are extremely important for early diagnosis of these infants.
Sepsis can cause hypothermia and poor feeding, but the two-week time makes this choice unlikely. Hirschsprung disease may cause chronic constipation and abdominal distension, but not the other findings. Botu¬lism can cause a flaccid paralysis and poor feeding, but the large fontanelles and umbilical hernia are not caused by this infection.
A term, 4200-g female infant is delivered via cesarean section because of cephalopelvic disproportion. The amniotic fluid was clear, and the infant cried almost immediately after birth. Within the first 15 min of life, however, the infants respiratory rate increased to 80 breaths per min, and she began to have intermittent grunting respirations. The infant was transferred to the level 2 nursery and was noted to have an oxy¬gen saturation of 94%. A chest radiograph showed fluid in the fissure, over aeriation, and prominent pulmonary vascular markings. Which of the following is the most likely diagnosis?
a. Diaphragmatic hernia
b. Meconium aspiration
c. Pneumonia
d. Idiopathic respiratory distress syndrome
e. Transient tachypnea of the newborn
The answer is e. Transient tachypnea of the newborn is usually seen after a nor¬mal vaginal or, especially, after a cesarean delivery. The condition is a result of retained fetal lung fluid. These patients have tachypnea, retractions, grunting, and sometimes cyanosis. The chest examination is usually nor¬mal; the chest radiograph demonstrates prominent pulmonary vascular markings with fluid in the fissures and hyperexpansion (flat diaphragms), Therapy is supportive, with maintenance of normal oxygen saturation. Resolution usually occurs in the first 3 days of life.
A routine prenatal ultrasound reveals a male fetus with meningomyelocele. The 24-year-old primigravid mother is told the infant will require surgery shortly after birth. You counsel her about the etiology of this defect and the risk of further pregnancies being similarly affected, and state which of the following?
a. The hereditary pattern for this condition is autosomal recessive
b. The prenatal diagnosis can be made by the detection of very low levels of alpha-fetoprotein in the amniotic fluid
c. Subsequent pregnancies are not at increased risk compared to the general pop¬ulation
d. Supplementation of maternal diet with folate leads to a decrease in incidence of this condition
e. Neither environmental nor social factors have been shown to influence the inci¬dence
The answer is d. Diseases that are due to defects in a sin¬gle gene are designated as autosomal or X-linked, depending on whether the affected gene is located on an autosome or an X chromosome. Geneti¬cally determined diseases that are multifactorial in origin (i.e., neural tube defects) do not conform to the Mendelian pattern of inheritance but exhibit a variable outcome that reflects the interaction between a particular geno¬type and an environment. The relatives of persons with diseases of multi-factorial origin have an increased risk of having similar abnormalities.
The recurrence risk for most single primary defects of multifactorial inheritance (e.g., neural tube defects) is increased with each child affected. This increased risk forms the basis for assuming that genetic factors play a role in the occurrence of these abnormalities. Other factors, such as race, sex, and ethnic and racial background, influence the frequency with which an abnormality of multifactorial inheritance occurs in relatives. The prena¬tal diagnosis of neural tube defects (anencephaly and meningomyelocele) can be made by the detection of elevated levels of a-fetoprotein in the amniotic fluid. To reduce the risk of neural tube defects, it is now recom¬mended that all women capable of becoming pregnant take 400 meg of folic acid daily.
A previously healthy full-term infant has several episodes of duskiness and apnea during the second day of life. Diagnostic considerations should include which of the following?
a. Hemolytic anemia
b. Congenital heart disease
c. Idiopathic apnea
d. Harlequin syndrome
e. Hypoglycemia
The answer is b. Idiopathic apnea is common in prema¬ture infants but is not expected in the full-term newborn. When apnea occurs in the term infant, there is almost always an identifiable cause. Sep¬sis, gastroesophageal reflux, congenital heart disease, seizures, hypoglycemia, and airway obstruction can cause apnea in term newborns. Harlequin syndrome is a transient change in the skin color of the otherwise asymptomatic newborn (usually preterm) in which the dependent side of the entire body turns red while the upper side remains pale.
The signs and symptoms of meningitis in an infant can be different than those in an adult. Which of the following signs and symptoms of meningitis is more helpful in an adult patient than in a 1-momh-old?
a. Lethargy
c. Vomiting
b. Jaundice
d. Brudzinki sign
c. Hypothermia
The answer is d. Neonatal sepsis, a clinical syndrome of systemic illness accompanied by bacteremia, often results in spread of infection to the meninges and other distant sites. The diagnosis of serious infection,, including meningitis, in a neonate is difficult because the signs and symp¬toms are subtle and nonspecific. They include lethargy; feeding problems including abdominal distention, vomiting, and diarrhea; temperature instability; respiratory distress or apnea; and jaundice. Nuchal rigidity and Kernig and Brudzinski signs are frequently not present in the neonate with meningitis.
A woman gives birth to twins at 38 weeks’ gestation. The first twin weighs 2800 g and has a hematocrit of 70%; the second twin weighs 2100 g and has a hematocrit of 40%. Which of the fol-lowing statements is correct?
a. The second twin is at risk for developing respiratory distress, cyanosis, and congestive heart failure
b. The first twin is more likely to have hyperbilirubinemia and convulsions
c. The second twin is at risk for renal vein thrombosis
d. The second twin probably has hydramnios of the amniotic sac
e. The second twin is likely to be pale, tachycardic, and hypotensive
The answer is b. Twin-to-twin transfusions occur in about 15% of monochorionic twins and commonly cause intrauterine death. This dis¬order should be suspected when the hematocrits of twins differ by more than 15 mg/dL. The donor twin is likely to have oligohydramnios, anemia, and hypovolemia with evidence of shock if the hematocrit is significantly reduced; the recipient twin is likely to have hydramnios and plethora and to be larger than the donor twin. A 20 percent difference in body weight may result. As the central venous hematocrit rises above 65%, infants can develop hyperviscosity, respiratory distress, hyperbilirubinemia, hypocalcemia, renal vein thrombosis, congestive heart failure, and convulsions.
Parents bring a 5-day-old infant to your office. The mother is O neg¬ative and was Coombs positive at delivery. The term child weighed 3055 g at birth and had measured a baseline hemoglobin of 16 gm/dl and a total serum bilirubin of 3 mg/dL. He passed a black tarlike stool within the first 24 h of life. He was discharged at 30 h of life with a stable axillary temperature of 36.5°C. Today the infant’s weight is 3000 g, his axillary temperature is 35°C (95°F), and he is jaundiced to the chest. Parents report frequent yellow, seedy stool. You withdraw labs and find his hemoglobin is now 14 gm/dL, and his total serum bilirubin is 13 mg/dL The change in which of the following parameters is of most concern?
a. Hemoglobin
b. Temperature
c Body weight
c. Bilirubin
d. Stool
The answer is b. There is loss of body weight of 1.5 to 2% per day for the first 5 days of life for a nor¬mal newborn infant as excessive fluid is excreted. One might think this would tend to cause hemoconcentration and produce an increase in hema¬tocrit, but, to the contrary, the hematocrit falls as an adaptation to an envi¬ronment of higher oxygen. Infants usually have several meconium stools during the first day or two of life, changing to soft yellow stools at 1 to 2 days of life. As the hematocrit falls, there is a corresponding increase in serum bilirubin that peaks around 3 to 5 days of life. Temperature should not change; temperature instability in a term infant is frequently a sign of serious infection
You are called to a delivery of a woman with no prenatal care, she is in active labor but has no history of amniotic rupture. The biophysical pro¬file done in the emergency center revealed severe oligohydramnios. When you get this infant to the nursery, you should carefully evaluate him for which of the following?
a. Anencephaly
b. Trisomy 18
c. Renal agenesis
d. Duodenal atresia
e. Tracheoesophageal fistula
The answer is c. It is generally presumed that duodenal atresia and tracheoesophageal fistula lead to hydramnios (polyhydramnios) by interference with reabsorption of swallowed amniotic fluid. Hydramnios is also associated with approximately 80% of infants who have trisomy 18. Approximately 50% of women with anencephalic fetuses have polyhy¬dramnios. Oligohydramnios occurs in association with congenital abnor-malities of the fetal kidneys or other parts of the genitourinary tract, such as renal agenesis or obstruction, that impede normal formation or excre¬tion of fetal urine.
Major causes of neonatal mortality include all of the following except:
a) Sudden infant death syndrome
b) Respiratory distress syndrome
c) Congenital malformations
d) Asphyxia
e) Sepsis
Answer is a
Sudden infant death syndrome (SIDS) usually does not occur in the first month of life. SIDS should not be confused with apnea of prematurity, because one is not related to the other. Apnea of prematurity occurs in the first month of life but does not contribute to mortality.
All of the following regarding the low-birthweight rate are true except:
a) Low birthweight contributes to neonatal mortality
b) The low birthweight mortality has decreased between 1981 and 2000
c) Low birthweight may be due to prematurity
d) Low birthweight may be due to intrauterine growth retardation
e) Low birthweight mortality is higher in black infants
The answer is b
Indeed the low-birthweight rate has increased during this interval. This can be explained only in part by a rising number of multiple births following reproductive technology advances.
All of the following are features of postneonatal mortality except:
a) Death between age 28 days and 1 yr
b) Caused in part by sudden infant death syndrome
c) Caused in part by bronchopulmonary dysplasia
d) Caused in part by premature rupture of membranes
e) Caused in part by trauma
The answer is d
Premature rupture of membranes (PROM) is associated with risks of early-onset sepsis and premature birth. Both should contribute to neonatal, not postnatal, mortality.
Historical family features that suggest risk in a current pregnancy include all of the following except:
a) A sibling with neonatal jaundice
b) Maternal history of urinary tract infection
c) Maternal age 19-35 yr
d) Maternal diabetes mellitus
e) Poor prenatal care
The answer is c
These are the most ideal ages for safe maternal reproduction. Adolescents and women over the age of 40 yr are considered high risk.
A 30-min-old term 3.4-kg baby, born after a spontaneous vaginal
delivery, is noted to have acrocyanosis. The most important next step is:
a) Perform a sepsis evaluation
b) Perform a CBC followed by a sepsis work-up if neutropenia is found
c) Keep the infant warm
d) Begin oxygen administration
e) Hold oral feedings until the acrocyanosis resolves
The answer is c: Acrocyanosis is normal at this time, and if the physical examination is also normal there is no risk of a serious underlying disorder. Warming is all that may be needed.
An African-American male is born after a normal pregnancy and
delivery with rupture of the fetal membranes for 3 hr and no fetal monitoring.
Immediately after birth, he is noted to have multiple small pustules over his chin and
neck that are on a pigmented macular base. The most likely diagnosis is:
a) Erythema toxicum
b) Pustular melanosis
c) Congenital herpes simplex, type II
d) Incontinentia pigmentosa
e) Herpes gestationalis
The answer is b
This is the classic presentation of this benign neonatal rash. The flaccid vesicles contain neutrophils, which is in contrast to the eosinophils seen in erythemia toxicum.
Deformational plagiocephaly is associated with all of the following except:
a) Breech presentation
b) Torticollis
c) Asymmetrical skull
d) Ear malalignment
e) Asymmetric face
The answer is a:
Vertex positioning with possible pressure of the developing skull by the mother’s bony pelvis contributes to this deformation
Bilateral and multiple retinal hemorrhages, presenting in the 6th week
of life in a lethargic neonate without any other physical findings, are most likely due
to:
a) Birth trauma
b) Nuchal cord
c) Vacuum extraction
d) Being large for gestational age
e) Child abuse
The answer is e
Child abuse is often associated with bilateral retinal hemorrhages, especially in what is called the “shaken baby syndrome.” Retinal hemorrhages may be noted immediately after birth. Nonetheless, they should all resolve by 1 mo of age.
A 3.5-kg female born following repeat cesarean section is noted by
the nurses to be grunting at 10 min of age. You come to see the baby and note that
the grunting has stopped, the respiratory rate is 36/min, the pulse oximetry reading
is 99%, and the child looks vigorous. The most appropriate next step is to:
a) Perform a sepsis evaluation
b) Obtain a chest film
c) Observe, and if grunting returns, admit to the neonatal intensive care unit
d) Begin surfactant therapy
e) Begin nasal continuous positive airway pressure
The answer is c
Grunting is common, particularly after a cesarean section without prior labor. If grunting persists beyond 30 min or if there are other signs of distress, the child should be evaluated for sepsis, respiratory distress syndrome (RDS), or congenital heart disease.
The best description of the Apgar score is that it:
a) Accurately predicts who will develop cerebral palsy
b) Assesses neonates in need of resuscitation
c) Accurately predicts a low umbilical cord pH
d) Is unaffected by maternal opiate pain relief
e) Accurately predicts neonates who will die in the neonatal period
Answer is b
The Apgar score helps to rapidly assess the need to resuscitate neonates after birth. Although it has some value in predicting neonatal mortality and cerebral palsy, it has a poor positive predictive value. Most children with cerebral palsy have had normal Apgar scores, while neonates with low Apgar scores do not universally get cerebral palsy.
Reproductive technologies are associated with all of the following pregnancy-related risks except a:
a) Higher risk for multiple fetuses
b) Higher risk for group B streptococcal sepsis
c) Higher risk for low birthweight
d) Higher risk for very low birthweight
e) Higher risk for congenital anomalies
Answer is b
Group B streptococci are not increased in these pregnancies. In addition to the risks of low birthweight (LBW) and very low birthweight (VLBW), as well as multiple-fetus
pregnancies, there is an increased risk of cerebral palsy, related in part to preterm birth.
Spontaneous uterine contraction monitoring is most useful to:
a) Accurately identify risk for very low birthweight
b) Determine risk of premature rupture of the membranes
c) Make diagnosis of abruptio placentae
d) Identify a short cervix
e) None of the above
Answer is e
Although monitoring for the rate of spontaneous uterine contractions before the onset of labor is popular, it has a poor sensitivity and a low positive predictive value for premature birth.
Oligohydramnios is associated with all of the following except:
a) Esophageal atresia
b) Pulmonary hypoplasia
c) Potter syndrome
d) Posterior urethral valves
e) Skeletal contractures
Answer is a
Esophageal atresia is most often associated with polyhydramnios, as are other upper intestinal obstructive lesions or disorders of fetal swallowing.
15. Fetal ultrasonography is of use for all of the following except:
a) Determining multiple fetuses
b) Identifying congenital heart disease
c) Determining the lecithin-sphingomyelin (L/S) ratio
d) Screening for trisomy 21
e) Predicting intrauterine growth retardation
Answer is c
The lecithin-sphingomyelin ratio (L/S) predicts pulmonary maturity and is a functional test that is based on maturation of the fetal type II pulmonary alveolar cells. Fetal ultrasonography may estimate fetal gestational age, and thus the potential for lung maturity, but it cannot ascertain the actual L/S ratio.
Fetal tachycardia may be caused by all of the following except:
a) Maternal lupus erythematosus
b) Maternal fever
c) Prematurity
d) Fetal anemia
e) Fetal supraventricular tachycardia
Answer is a
Maternal lupus may affect the fetal cardiac conduction system, and it produces bradycardia by the
development of an antibody immune-related process.
Immunologic diseases of the mother that may affect the fetus include all of the following except:
a) Systemic lupus erythematosus
b) Idiopathic thrombocytopenic purpura years after splenectomy
c) Myasthenia gravis
d) Graves disease
e) Multiple sclerosis
Answer is e
It is interesting that B might be a choice. However, splenectomy reduces the destruction of maternal platelet antibody complexes by the spleen, thus increasing the mother’s platelet count. Splenectomy does not reduce the IgG antiplatelet antibodies, which can readily cross the placenta.
21. An important
An important influence on the effects of cigarette smoking on fetal
growth restriction is:
a) Brand of cigarettes
b) Altitude
c) Polymorphisms of hydrocarbon-metabolizing enzymes
d) Filtered versus non-filtered cigarettes
e) Educational level of the mother
Answer is c
Polyhydric aromatic hydrocarbons may alter fetal growth. It is very possible that enzymes that metabolize these to either toxic or-more likely-nontoxic metabolites modify the impact of the fetal growth restriction due to cigarette smoking.
Cordocentesis (percutaneous umbilical blood sampling) is useful for all of the following except:
a) Measuring fetal hemoglobin
b) Determining fetal genome
c) Performing fetal cardiac catheterization
d) Measuring antibody titers
e) Measuring fetal pH
Answer is c
This is future shock! Although one case of fetal valvotomy has been reported, it was not performed via PUBS but via fetal cardiac puncture.
Amnion nodosum is associated with all of the following except:
a) Oligohydramnios
b) Fetal Candida infection
c) Renal agenesis
d) Pulmonary hypoplasia
e) Flattened facies
Answer is b
Fetal-placental amniotic Candida infection may demonstrate nodules on the umbilical cord and a neonatal macular-papular, erythematous, disseminated rash.
The incidence of twins detected at 40 wk of gestation is characterized by all of the following except:
a) Family history of monozygotic twins
b) Fetal mortality of twins noted at 12-wk of gestation
c) Family history of dizygotic twins
d) Race
e) Infertility treatment
Answer is a
Monozygotic twinning has little or no genetic determination. Many twins detected at 12 wk of gestation are singletons at 40 wk owing to fetal twin demise. This process
may affect the surviving twin (e.g., incidence of cerebral palsy
is higher in the surviving twin).
Very-low-birthweight infants are best described as:
a. Predominantly growth restricted
b. Predominantly premature
c. Predominantly post dates
d. The result of maternal illness
e. The result of placental infarction
The answer is b
Some VLBW infants may have growth restriction, but almost all infants weighing <1500 g are premature.
Excessive intravenous fluid intake in very-low-birthweight infants is associated with:
a) High BUN level
b) Bronchopulmonary dysplasia
c) Hyperbilirubinemia
d) Hypernatremia
e) Oliguria
The answer is b
Both bronchopulmonary dysplasia (BPD) and a patent ductus arteriosus (PDA) are associated with excessive intravenous fluids in premature infants. Answers A, C, D, or E are some reasons why fluid intakes may be increased, but at the same time, the infant is placed at risk for BPD or a PDA.
Hypotension in a newborn suggests all of the following except:
a) Fetal-maternal hemorrhage
b) Hypoplastic left heart syndrome
c) Adrenogenital syndrome
d) Tension pneumothorax
e) Jaundice
The answer is e
Answers A to D, other congenital heart defects, postnatal hemorrhage (CNS, liver, spleen), and sepsis can all produce hypotension.
A 900-g infant with respiratory distress syndrome has persistent hypotension despite administration of fluid boluses and dopamine. Epinephrine also fails to elevate the blood pressure. The next step to manage the blood pressure is to administer:
a) Albumin
b) Hydrocortisone
c) Hypertonic saline
d) Norepinephrine
e) Dobutamine
The answer is b
Many studies have demonstrated the value of administering hydrocortisone (Solu-Cortef) to VLBW infants who respond poorly to fluid boluses and dopamine.
A 1-day-old infant born to a diabetic mother is noted to have tremors of all extremities while awake. They disappear during sleep and are not associated with eye movements. The glucose and calcium levels are normal. The most likely diagnosis is:
a) Hypoglycemia
b) Kernicterus
c) Renal vein thrombosis
d) Caudal regression syndrome
e) Jitteriness
The answer is e
This is a classic problem in infants of diabetic mothers (IDMs). All jittery children should be examined carefully and concern for a seizure considered. If the child appears normal and has no abnormal eye movements, and if the motion is sensory-dependent, is only present during active wakefulness, and is stopped by simple pressure, it is probably not a seizure. Nonetheless, it could be associated with a serious problem such as hypocalcemia, hypoglycemia, or opiate withdrawal. IDMs are often more jittery than other infants but are also at risk for hypoglycemia and hypocalcemia.
A 5-day-old, large-for-gestational-age, 4,500-g boy has a bilirubin
level of 21 mg/dL. There is no anemia or polycythemia, but on examination he has a
large cephalohematoma. The next therapeutic activity should be to:
a) Aspirate the hematoma
b) Perform an incision and drainage of the hematoma
c) Undertake prophylactic blood transfer
d) Administer phototherapy
e) Perform exchange transfusion
The answer is d
Phototherapy is clearly indicated. Aspiration or incision and drainage (I + D) should not be done to manage a cephalohematoma.
Routine head ultrasonography in infants <1,500 g to detect intracranial hemorrhage is best described as:
a) Performed between 7 and 14 days and at 36-40 wk
b) Performed for anemia
c) Performed for seizures
d) Performed at birth and at 40 wk
e) Performed between 7 and 14 days and at 1 yr of age
The answer is a
In addition, nonroutine ultrasonography should be performed for symptoms of intraventricular hemorrhage (IVH) and for the follow-up of abnormalities noted on the first ultrasound.
The management of post-hemorrhagic hydrocephaly includes all of the following except:
a) Serial head circumferences
b) Serial head ultrasound examinations
c) External ventricular drainage
d) Ventricular-peritoneal shunt
e) Repeated lumbar punctures
The answer is e
Although repeat lumbar puncture (LP) is often done, most physicians do not believe that they avoid the need for a ventricular peritoneal (VP) shunt. Most cases of dilated ventricles after IVH do not necessitate later placement of a shunt.
A 12-day-old, large-for-gestational-age infant is noted to have Erb palsy. You should do all of the following except:
a) Refer for immediate neuroplasty
b) Refer for physical therapy
c) Reassure the family
d) Determine if the clavicle is fractured
e) Look for additional nerve involvement (phrenic)
The answer is a
Most Erb palsies resolve rapidly with immobilization, rehabilitation, and positioning. If there is no improvement between 3-6 mo, a referral for surgical evaluation
is indicated.
25. A term female is born by spontaneous vaginal delivery to a primiparous woman who received two doses of meperidine 30 min and 2 hr prior to an abrupt delivery. The baby is apneic and limp. The most important, immediate management is to:
a) Administer naloxone in the umbilical vein
b) Perform bag-mask ventilation
c) Administer naloxone in the endotracheal tube
d) Begin chest compressions
e) Obtain a cord pH
The answer is b
Apnea from any cause is treated with securing a patent airway and instituting ventilation. If bag mask ventilation is ineffective, endotracheal intubation should be performed.
The existing evidence from randomised controlled trials is insufficient to determine whether naloxone confers any important benefits to newborn infants with cardiorespiratory or neurological depression that may be due to intrauterine exposure to opioid. Given concerns about the safety of naloxone in this context, it may be appropriate to limit its use to randomised controlled trials that aim to resolve these uncertainties
26. Successful ventilation is determined by all of the following except:
a) Zero reading of end-tidal CO2 measurement
b) Pink color
c) Rising heart rate
d) Symmetric breath sounds
e) Good chest rise
.
The answer is a
Indeed, one expects that with a successful intubation and ventilation, the end-tidal (exhaled) CO2 will rise dramatically. This is now used in many neonatal resuscitations
27. A 3-wk-old, 32-wk-gestation, premature infant has sudden onset of apnea and bradycardia that occurs repeatedly. On examination, the infant appears to be asleep, and mild abdominal distention is noted. The approach to the patient’s management is best performed by:
a) Considering this a case of apnea of prematurity
b) Being concerned about a potential medical condition
c) Obtaining a head ultrasound study
d) Doing an echocardiogram
e) Starting caffeine
The answer is b
A previously healthy “gainer and groaner” should not develop apnea of prematurity at 3 wk of age and at a corrected age of 35 wk. Sudden onset and repeated episodes strongly suggest an underlying illness. The abdominal distention also suggests an ileus or necrotizing enterocolitis.
28. A 32-wk gestational age infant develops grunting, flaring, and retraction after birth. He requires 50% oxygen (O2) by hood to keep his oxygen saturation above 95%. The next step in management, if he requires more O2, is to:
a) Institute inhalation of nitric oxide
b) Perform a blood transfusion
c) Begin nasal continuous positive airway pressure (CPAP)
d) Begin penicillin infusion
e) Begin dexamethasone infusion
The answer is c
Nasal CPAP is often quite effective when larger premature infants develop respiratory distress syndrome
29. In addition to administration of CPAP, immediate approaches to the patient described in the previous Question, would include all of the following except:
a) Head ultrasonography
b) Blood culture
c) Chest x-ray
d) Complete blood count
e) Blood gas analysis
The answer is a
In a neonate with no signs of intraventricular hemorrhage (IVH) and on the first day of life, one does not need a head ultrasound. The others (B-E) are important to look for other causes of respiratory distress and prepare to treat empirically for the possibility of early-onset group B
streptococcal pneumonia.
20. A 3-day-old, 790-g female infant had been ventilated for respiratory
distress syndrome and was being weaned effectively from the ventilator. Today she
is noted to have an active precordium, bounding pulses, and hypoxia with
hypercarbia. All of the following should be done to evaluate and manage her except:
a) Administer prostaglandin E2
b) Obtain a chest film
c) Obtain an echocardiogram
d) Restrict fluids
e) Treat the hypoxia
Answer is a
The possibility of a duct dependent cyanotic heart disease is unlikely, he mostly has PDA, chest infection is a possibility
21. The immediate treatment of choice for the patient described in the previous Question is administration of:
a) Digoxin
b) Dobutamine
c) Dexamethasone
d) Indomethacin
e) Vitamin E
The answer is d
Indomethacin, and perhaps ibuprofen when approved by the FDA, inhibits prostaglandin synthesis,
facilitating medical closure of a PDA. If medical treatment is unsuccessful, surgical ligation is performed.
22. The differential diagnosis of persistent pulmonary hypertension of the newborn includes all of the following except:
a) Polycythemia
b) Total anomalous pulmonary venous return
c) Meconium aspiration pneumonia
d) Hypoglycemia
e) Renovascular hypertension
The answer is e
Systemic hypertension may be a complication of treating persistent pulmonary hypertension of the newborn (PPHN) by ECMO or from umbilical artery catheterization.
The answer is b
All infants with bile stained emesis should be evaluated radiologically. A plain film or KUB film is not sufficient to detect all abnormalities but should be done before a barium intestinal series.
The answer is e
Systemic hypertension may be a complication of treating persistent pulmonary hypertension of the newborn (PPHN) by ECMO or from umbilical artery catheterization.
24. Meconium plug is associated with all of the following except:
a) Hypermagnesemia
b) Infants of diabetic mothers
c) Cystic fibrosis
d) Hirschsprung disease
e) Prematurity
The answer is e
Although prematurity may cause delayed passage of meconium (NPO, immature colonic function), it is not associated with a higher incidence of meconium plugs.
25. All of the following are true about necrotizing enterocolitis except:
a) Epidemics occur
b) Prematurity is the greatest risk factor
c) Human milk reduces the incidence
d) Surgeons need not be involved until there is evidence of intestinal perforation
e) Pneumatosis intestinalis is an important diagnostic radiologic sign
The answer is d
Surgeons should be involved early to help in the management and to discuss indications for surgery, including evidence of perforation and failure to respond to medical
management. Placement of an abdominal drain with peritoneal lavage may also help in the management of a sick VLBW infant.
26. High risk factors for neonatal jaundice include all of the following except:
a) Neonatal polycythemia
b) A sibling with jaundice
c) Poor enteral intake
d) Asian heritage
e) Post dates
The answer is e
Postmature infants have a lower incidence of jaundice unless polycythemia is present. Other risk factors for jaundice include hemolysis, Gilbert disease, breast-feeding, prematurity, diabetic mother, bruising, intestinal obstruction, hypothyroidism, and diseases producing cholestatic disorders.
27. A 3-wk-old breast-fed infant has deepening jaundice. On physical examination, the liver is 3 cm below the right costal margin. The most important laboratory test in this child at this time is:
a) Serum ceruloplasmin determination
b) Direct and total bilirubin level
c) Hepatic ultrasonography
d) Complete blood count
e) Urine urobilinogen determination
The answer is b
Until this test is done, it is unknown whether the infant has cholestatic or indirect hyperbilirubinemia. In this child, the total bilirubin was 20 mg/dL and the direct was 10
mg/dL. She had biliary atresia.
28. A term female infant is born with Apgar scores of 9 and 9. At 15 hr of age, she is noted to be pale. The vital signs reveal tachycardia; there is no hepatosplenomegaly or jaundice. The family history is not contributory, and the review of the labor and delivery do not reveal any sources of blood loss. Her hematocrit at 16 hr of age is 30%. The reticulocyte count is 15%, whereas the platelet and WBC counts are normal, as is the blood smear. The bilirubin is 2 mg/dL.
The next important step in her evaluation is to do:
a) Red blood cell fragility test
b) Coombs test
c) Kleihauer-Betke test
d) Apt test
e) Serum ferritin determination
The answer is c
The Kleihauer-Betke test is performed on maternal blood and tests for the presence of fetal hemoglobin containing erythrocytes from a fetal-to-maternal transfusion. A low bilirubin suggests that there is no hemolysis, and a normal examination, except for tachycardia, suggests no internal blood loss. Fetal-to-maternal bleeding can be chronic or acute.
29. The role of recombinant human erythropoietin in the management of very low birthweight is best defined as:
a) Providing prophylaxis in jaundiced infants
b) Providing prophylaxis in infants only <750 g
c) Preventing of bronchopulmonary dysplasia
d) Preventing of retinopathy of prematurity
e) None of the above, it has no routine role
The answer is e
Recombinant human erythropoietin (rHuEpo) has not consistently demonstrated benefit. Indeed, in older patients, antibodies to erythropoietin may develop, which will bind to endogenous erythropoietin, thus producing a severe anemia. It may increase risk of retinopathy of prematurity
Matching: ABO vs. Rh hemolytic disease.
Kernicterus
A. ABO
B. Rh
C. Both
D. Neither
C. Both
Jaundice on day 1
A. ABO
B. Rh
C. Both
D. Neither
C. Both
Increasing severity with subsequent pregnancies
A. ABO
B. Rh
C. Both
D. Neither
B. Rh
Anemia
A. ABO
B. Rh
C. Both
D. Neither
C. Both
Intracranial hemorrhage
A. ABO
B. Rh
C. Both
D. Neither
D. Neither
In utero treatment
A. ABO
B. Rh
C. Both
D. Neither
B. Rh
Preventable with immunoglobulin
A. ABO
B. Rh
C. Both
D. Neither
B. Rh
Iron deficiency
A. ABO
B. Rh
C. Both
D. Neither
D. Neither
Hydrops fetalis
A. ABO
B. Rh
C. Both
D. Neither
Explanation: B. Rh
Hepatosplenomegaly
A. ABO
B. Rh
C. Both
D. Neither
Explanation: B. Rh
Polycythemia is associated with all of the following except:
a) Trisomy 21
b) Donor twin in a twin-twin transfusion
c) Neonatal Graves disease
d) Adrenogenital syndrome
e) Infant of diabetic mother
Answer is b
The recipient twin develops polycythemia.
Additional risks for polycythemia include intrauterine growth retardation, postmature infants, Beckwith-Wiedemann syndrome, and being born at high altitude.
A term fetus is noted to have bilateral dilation of the renal pelvis and ureter. Amniotic fluid volume is adequate, and the bladder size looks normal. After birth, the child voids spontaneously within 24 hr. The most appropriate diagnostic approach to this child is:
a) Renal ultrasound study at birth and at 1 mo
b) Renal ultrasound study at birth
c) Renal ultrasound study at 1 mo
d) Intravenous pyelogram
e) None of the above. no diagnostic evaluation is needed because he voided spontaneously within 1 day of birth
Answer is a
Because fluid intake and urine flow may be reduced in the first day or two after birth, an obstruction may be missed if only one ultrasound is performed. Therefore, it is recommended to repeat the ultrasound examination at 1 mo of age.
A 2-mo-old African-American term male has a seizure at home. In the emergency department he seems awake but has tremors and feels rigid. He is being breast-fed by a mother who is a strict vegetarian. Prior to this, his growth and development was normal. He feeds often and well compared with her other children who are 18 mo, 5 yr, and 10 yr old. The most likely diagnosis is:
a) Nesidioblastosis
b) Intracranial hemorrhage
c) Hypoglycemia
d) Pyridoxine deficiency
e) Hypocalcemia
Answer is e
This infant has late neonatal onset tetany, which is usually due to ingesting cow’s milk with a very high phosphate content. However, some infants breast-fed by a mother at nutritional risk may develop hypocalcemia or, later in infancy, rickets. The patient responded to intravenous calcium
and was sent home on supplemental vitamin D. Infants with hypocalcemia often have only peripheral neuromuscular signs and appear alert during what appears to be a seizure.
The best method to identify prenatal drug misuse is:
a) Careful maternal history
b) Meconium testing
c) Testing of neonatal urine
d) Abstinence withdrawal score
e) Maternal urine testing
Answer is b
Meconium drug testing has become the standard and reflects long-term drug misuse at a site of drug excretion and storage. Urine test results may vary, depending on the timing between drug intake and testing and the dose.
The best approach to prevent congenital anomalies in infants of diabetic mothers is to:
a) Discontinue insulin and begin glyburide
b) Provide continuous insulin infusion during labor and delivery
c) Switch from an oral hypoglycemic agent to insulin until 36-wk of gestation
d) Do serial glucose tolerance tests
e) Maintain periconceptional tight control of maternal blood glucose levels
Answer is e
Periconception control and control during the early period of organogenesis help reduce the risk of
congenital malformations. Common anomalies affect the heart and musculoskeletal system.
A 4.5-kg baby girl of a mother with classic diabetes develops grunting after cesarean delivery. At 40 hr of age, the baby has tachycardia, tachypnea, cyanosis, and a normal blood glucose level. A chest x-ray reveals cardiomegaly, and you believe that the infant is in heart failure. A fetal
echocardiogram does not reveal any evidence of congenital heart anomalies. The
therapeutic agent of choice for this infant is:
a) Digoxin
b) Dobutamine
c) Propranolol
d) Epinephrine
e) Dopamine
.
Answer is c
This infant has cardiomyopathy associated with infants of diabetic mothers. This is a form of ventricular septal hypertrophy that can obstruct the left ventricular outflow tract and cause heart failure and poor cardiac output. Inotropic agents that increase the force of ventricular muscle contraction exacerbate the obstruction and are contraindicated
A newborn boy is noted on physical examination to have a unilateral cleft lip, bilateral postaxial polydactyly, microcephaly, intrauterine growth retardation, and micropenis. You are unable to make a specific overall diagnosis. What is the most appropriate initial diagnostic test for this patient?
a) Quantitative urinary organic acid measurement
b) Plasma amino acid determination
c) Karyotype
d) 17-hydroxyprogesterone assay
e) Urinary mucopolysaccharide screen
Answer is c
This constellation of findings suggests a chromosomal abnormality. Multiple malformations however may be of unknown etiology. Metabolic disorders rarely cause multiple malformations but may result in normal formation with subsequent changes, such as coarsening of the facial features
or microcephaly or macrocephaly.
A 2-day-old female infant is noted to have a bilateral cleft lip and cleft palate. Based on a careful physical examination, you determine that she is otherwise completely normal from a structural standpoint. Neither parent has a cleft lip or cleft palate. Upon careful questioning of the mother, you determine that she was not taking any medications during her pregnancy. Based on a careful family history, you further determine that no other family member is affected. Which of the
following recurrence risk figures would you give this baby’s parents?
a) 25%
b) 0%
c) 50%
d) 3-5%
e) 75%
Answer is d
Recurrence risks of what is traditionally thought to be a polygenetic disorder are between 3-5%. This is seen in congenital heart disease and other single-organ malformations without a positive family history.
A 1-day-old male infant, born to an 18-yr-old primigravida female, is noted to have multiple joint contractures and to be microcephalic. The pregnancy was complicated by polyhydramnios. Which of the following is the most likely explanation for these clinical findings?
a) A serious defect in CNS development
b) Deformational defects associated with a primigravida mother
c) Uterine fibroids leading to deformational defects
d) A chromosomal disorder associated with deformational defects
e) Renal agenesis
Answer is a
Polyhydramnios suggests obstruction of the gastrointestinal tract or disorders of swallowing due to poor CNS function in utero. Contractions suggest poor motor activity
in the fetus or being fixed in one position (this is unlikely in the presence of polyhydramnios).
The most effective way to prevent early onset of neonatal group B streptococcal sepsis is:
a) Screen all pregnant women for group B streptococci
b) Employ higher risk criteria
c) Give intramuscular penicillin to all newborns
d) Institute tocolysis to prevent preterm birth
e) There is no effective prevention
Answer is a
Most studies suggest that screening for group B Streptococcus in the mother and then selectively treating affected women will reduce the risk of neonatal sepsis.
A 30-wk-old average-for-gestational-age male infant is born after 24 hr of ruptured membranes and another 10 hr of labor. After birth, he develops grunting, flaring, and restrictions, as well as cyanosis. He requires intubation, ventilation, and surfactant therapy. Despite these measures he is still hypoxic and hypotensive. You should now:
a) Do a blood culture and await results
b) Do a Gram stain of the tracheal fluid
c) Do a blood culture and begin intravenous antibiotics
d) Begin intravenous antibiotics
e) Obtain blood and CSF for culture and begin intravenous Antibiotics
Answer is c
Premature infants with respiratory distress syndrome (RDS) may look indistinguishable from infants with early-onset sepsis. Waiting for culture results in this situation would be dangerous, as the infant will die without therapy before the culture is reported positive. A lumbar puncture is not wise in this case, as the infant is hypotensive and hypoxic; the blood culture is the only needed initial culture. If the blood culture is positive, when the patient’s condition becomes more
stable, a lumbar puncture can be performed.
Matching: Malformations
A. Rubinstein-Taybi syndrome
B. Trisomy 21
C. Williams syndrome
D. Pierre Robin syndrome
E. Prader-Willi syndrome
F. VATER syndrome
G. None of the above
Deletion of elastin allele
A. Rubinstein-Taybi syndrome
B. Trisomy 21
C. Williams syndrome
D. Pierre Robin syndrome
E. Prader-Willi syndrome
F. VATER syndrome
G. None of the above
Answer is : C. Williams syndrome
Microdeletion 16p13.3
A. Rubinstein-Taybi syndrome
B. Trisomy 21
C. Williams syndrome
D. Pierre Robin syndrome
E. Prader-Willi syndrome
F. VATER syndrome
G. None of the above
Answer is A. Rubinstein-Taybi syndrome
Deletion of band q11-12
A. Rubinstein-Taybi syndrome
B. Trisomy 21
C. Williams syndrome
D. Pierre Robin syndrome
E. Prader-Willi syndrome
F. VATER syndrome
G. None of the above
Answer is: E. Prader-Willi syndrome
Endocardial Cushing effects
A. Rubinstein-Taybi syndrome
B. Trisomy 21
C. Williams syndrome
D. Pierre Robin syndrome
E. Prader-Willi syndrome
F. VATER syndrome
G. None of the above
Answer is: B. Trisomy 21
Mandibular hypoplasia
A. Rubinstein-Taybi syndrome
B. Trisomy 21
C. Williams syndrome
D. Pierre Robin syndrome
E. Prader-Willi syndrome
F. VATER syndrome
G. None of the above
Answer is: D. Pierre Robin syndrome
Tracheoesophageal fistula
A. Rubinstein-Taybi syndrome
B. Trisomy 21
C. Williams syndrome
D. Pierre Robin syndrome
E. Prader-Willi syndrome
F. VATER syndrome
G. None of the above
Answer is: F. VATER syndrome
Anal atresia
A. Rubinstein-Taybi syndrome
B. Trisomy 21
C. Williams syndrome
D. Pierre Robin syndrome
E. Prader-Willi syndrome
F. VATER syndrome
G. None of the above
Answer is: F. VATER syndrome
Radial hypoplasia
A. Rubinstein-Taybi syndrome
B. Trisomy 21
C. Williams syndrome
D. Pierre Robin syndrome
E. Prader-Willi syndrome
F. VATER syndrome
G. None of the above
Answer is: F. VATER syndrome
An infant has the following findings at 5 min of life: pulse, 130/min; cyanotic hands and feet; good muscle tone; and a strong cry and grimace. This infant’s Apgar score is:
a) 7
b) 8
c) 9
d) 10
Answer is c:
One point in the Apgar score is taken off for color.
Which of the following is most appropriate for treating hyperbilirubinemia (11.2 mg/dL) in a 3-wk-old, breast-fed infant with normal growth and development?
a) Phototherapy
b) Exchange transfusion
c) Phenobarbital
d) None of the above
Answer is d
No treatment is necessary for the infant described in the question, assuming normal growth and
development.
A 2-wk-old infant is brought to the emergency room in coma with retinal hemorrhages and severe pallor. He was born at home and was first seen by a physician at 10 days of age and placed on amoxicillin for otitis media. His diet is breast milk. The day before admission, his parents took him in a four-wheel-drive vehicle on a hot day over a rough road in the mountains. Seizures began 8 hr later, and his condition steadily deteriorated for the next 16 hr. He oozes blood from all
venipuncture sites. Diagnostic tests should include all of the following except:
a) Coagulation studies
b) Skeletal survey
c) CT scan
d) Complete blood count
e) Lumbar puncture
Answer is e
Lumbar puncture is contraindicated in a child with intracranial hemorrhage
Immediate therapy for the infant described in the previous Question should
include administration of:
a) Vitamin A
b) Vitamin B6 (pyridoxine)
c) Vitamin C
d) Vitamin E
e) Vitamin K
The answer is e
The most likely diagnosis for the child described in Question 149 is:
a) Pyridoxine deficiency
b) Severe scurvy
c) Hemorrhagic disease of the newborn
d) Child abuse
e) Hypervitaminosis A
Answer is c
The death of the infant described in the previous Questions could have been prevented by which one of the following measures:
a) Administration of vitamin K at birth
b) Home visitor services
c) Discontinuance of antibiotics
d) Proper use of an infant seat
e) An air conditioner
The answer is A
This infant has a case of severe vitamin K deficiency-hemorrhagic disease of the newborn. The next most likely diagnosis is child abuse; most infants in coma with retinal
hemorrhages have been shaken, and a skeletal survey thus is appropriate. The combination of home delivery (no vitamin K administered), breast-feeding (low vitamin K content), and the amoxicillin treatment that eliminated normal intestinal bacterial synthesis of vitamin K led to the tragic demise of an otherwise normal infant. Of all possible preventive measures, administration of vitamin K at birth would have been the most effective.
A 3,600-g, breast-fed, white girl with a gestational age of 42 wk is noted to have persistent hyperbilirubinemia at 2 wk of age. On physical examination, the infant is found to have not gained weight since birth; she has decreased tone, an umbilical hernia, and an anterior fontanel measuring 4 x 6 cm. The most likely diagnosis is:
a) Crigler-Najjar syndrome
b) Gilbert disease
c) Biliary atresia
d) Hypothyroidism
e) Galactosemia
The answer is d
Hypothyroidism was confirmed by the late arrival of the newborn screening results, indicating high TSH and low T4 levels. Treatment with thyroxine improved the jaundice and the other signs. The hyperbilirubinemia is indirect (unconjugated). Crigler-Najjar syndrome is a possibility and is
either autosomal dominant or recessive (check the family history). However, there are signs other than jaundice that suggest another disease. Biliary atresia is always a concern in infants with delayed clearance of jaundice or worsening jaundice after 2 wk of life. The hyperbilirubinemia is
predominantly direct (conjugated). Galactosemia should be considered, especially in the presence of hypoglycemia, direct reacting jaundice, hepatomegaly, or ascites
A 4-wk-old, A-positive, African-American infant (former 40-wk gestational- age) was born to an O-positive mother and experienced hyperbilirubinemia, which required 2 days of phototherapy in the newborn nursery after birth. The infant appears apathetic and demonstrates pallor, a grade 2/6 systolic ejection murmur, and a heart rate of 175 beats/min. The most likely diagnosis is:
a) Anemia of chronic disease
b) Cholestasis secondary to neonatal hepatitis
c) Hereditary spherocytosis
d) Sickle cell anemia hemolytic crisis
e) ABO incompatibility with continued hemolysis
The answer is e
Jaundice usually resolves in all infants with hyperbilirubinemia due to ABO incompatibility in the first week of life. Nonetheless, the hemolysis continues without evidence
of jaundice because the liver can now excrete the bilirubin load. Late-onset anemia must be watched for and treated with a packed red blood cell transfusion if the infant is symptomatic.
Hereditary spherocytosis is a possibility but is relatively rare. A thorough family history and examination of the child’s and parents’ blood smear are helpful (because most cases of
spherocytosis are inherited as an autosomal dominant trait). Sickle cell anemia hemolytic crisis is not encountered this early in life because a considerable amount of fetal hemoglobin remains; thus, there are few -chains to sickle.
Matching: Congenital infections
A. CMV
B. Varicella
C. Enteroviruses
D. Rubella
E. Parvovirus
F. Toxoplasmosis
G. Trypanosoma cruzi
Megaesophagus
A. CMV
B. Varicella
C. Enteroviruses
D. Rubella
E. Parvovirus
F. Toxoplasmosis
G. Trypanosoma cruzi
Answer: G. Trypanosoma cruzi
Patent ductus arteriosus
A. CMV
B. Varicella
C. Enteroviruses
D. Rubella
E. Parvovirus
F. Toxoplasmosis
G. Trypanosoma cruzi
Answer: D. Rubella
Cerebral periventricular calcifications
A. CMV
B. Varicella
C. Enteroviruses
D. Rubella
E. Parvovirus
F. Toxoplasmosis
G. Trypanosoma cruzi
Answer: A. CMV
Limb hypoplasia
A. CMV
B. Varicella
C. Enteroviruses
D. Rubella
E. Parvovirus
F. Toxoplasmosis
G. Trypanosoma cruzi
Answer: B. Varicella
Myocarditis
A. CMV
B. Varicella
C. Enteroviruses
D. Rubella
E. Parvovirus
F. Toxoplasmosis
G. Trypanosoma cruzi
Answer: C. Enteroviruses
Fetal anemia
A. CMV
B. Varicella
C. Enteroviruses
D. Rubella
E. Parvovirus
F. Toxoplasmosis
G. Trypanosoma cruzi
Answer: E. Parvovirus
Hydrocephalus
A. CMV
B. Varicella
C. Enteroviruses
D. Rubella
E. Parvovirus
F. Toxoplasmosis
G. Trypanosoma cruzi
Answer:F. Toxoplasmosis, (
Matching:Maternal drugs and the fetus and newborn
A. Neonatal heart failure
B. Oligohydramnios
C. Spina bifida
D. Arthrogryposis
E. Facial-ear anomalies
F. Goiter
A. Neonatal heart failure
B. Oligohydramnios
C. Spina bifida
D. Arthrogryposis
E. Facial-ear anomalies
F. Goiter
Accutane
A. Neonatal heart failure
B. Oligohydramnios
C. Spina bifida
D. Arthrogryposis
E. Facial-ear anomalies
F. Goiter
answer: E. Facial-ear anomalies
Blue cohosh herbal tea
A. Neonatal heart failure
B. Oligohydramnios
C. Spina bifida
D. Arthrogryposis
E. Facial-ear anomalies
F. Goiter
answer: A. Neonatal heart failure
Iodide
A. Neonatal heart failure
B. Oligohydramnios
C. Spina bifida
D. Arthrogryposis
E. Facial-ear anomalies
F. Goiter
answer: F. Goiter
Ibuprofen
A. Neonatal heart failure
B. Oligohydramnios
C. Spina bifida
D. Arthrogryposis
E. Facial-ear anomalies
F. Goiter
answer: B. Oligohydramnios
Valproate
A. Neonatal heart failure
B. Oligohydramnios
C. Spina bifida
D. Arthrogryposis
E. Facial-ear anomalies
F. Goiter
answer: C. Spina bifida
Misoprostol
A. Neonatal heart failure
B. Oligohydramnios
C. Spina bifida
D. Arthrogryposis
E. Facial-ear anomalies
F. Goiter
answer: D. Arthrogryposis,
A term baby of an uncomplicated pregnancy is born limp, cyanotic, and apneic after a difficult vaginal delivery. Possible considerations in the differential diagnosis include all of the following except:
a) Prolapsed umbilical cord
b) Central nervous system trauma
c) Administration of morphine to the mother
d) Klumpke paralysis
e) Administration of local anesthetic into the fetal scalp
Answer is d
Klumpke paralysis involves injury to the 7th and 8th cervical nerves and the 1st thoracic nerve. It is usually unilateral, due to traction injury of the brachial plexus.
After intubation and resuscitation, the patient in the previous Question remains limp but appears aware and looks around, although the baby does not cry when the toes are pinched. The most likely diagnosis is:
a) Congenital botulism
b) Narcotic overdose
c) Transection of the spinal cord
d) Congenital myasthenia gravis
e) Neurosyphilis
The answer is c
Transection of the spinal cord may occur in vertex and breech positions and may be noted with normal vertebral body anatomy. It would manifest as in this patient, and also with shock, hypothermia, and bowel and bladder dysfunction. With time, hypotonia resolves into hypertonia and hyperreflexia.
A 2,100-g infant of a diabetic mother experiences seizures on the first day of life. Pregnancy was complicated by severe diabetic vasculopathy and placental insufficiency, with the development of late decelerations and the need for emergency caesarean section. The Apgar scores were 8 and 9 at 1 and 5 minutes, respectively. During the first 6 hr of life, the child was well and tolerated formula feedings well. Jitteriness developed at 10 hr of age and progressed to tonic-clonic seizures at 18 hr of age. Laboratory studies revealed blood glucose of 80 mg/dL and
calcium of 6.0 mg/dL. Thereafter, calcium gluconate (10%), 2 mL/kg, was given repeatedly without affecting the frequency of seizures. The most appropriate step to manage this infant’s condition is to:
a) Administer glucose
b) Administer pyridoxine
c) Administer 1,35-dihydroxyvitamin D
d) Administer magnesium sulfate
e) Administer lorazepam
The answer is d
Infants of diabetic mothers (IDM) often experience hypocalcemia within 24-48 hr after birth, due in part to an attenuated parathyroid gland response to reduced calcium levels. The IDM also develops hypomagnesemia, which, if uncorrected, results in persistent hypocalcemia due to
the dependency of parathyroid function on magnesium.